Chapter 11: Pricing Strategy, MKGT 351 chpt 20, Marketing 351: ch 21 Setting Prices, Chapter 20 Quiz Questions, MKT 300 Exam 3- Ch. 12, Chapter 19 Marketing Final, MKT321_CH20, Chap 19 Practice Questions, Foundations of Marketing Ch. 15, Mktg chapter...

Pataasin ang iyong marka sa homework at exams ngayon gamit ang Quizwiz!

A market share objective

can be used effectively whether total industry sales are rising or falling.

Pricing the basic product in a product line low while pricing related items at a higher level is called

captive pricing

A ___________ is a retail establishment that combines a supermarket and a discount store in one location. a. superstore b. department store c. category killer d. hypermarket e. megamarket

d

A narrow product mix with a deep product line would most likely be carried by a. mass merchandisers. b. supermarkets. c. discount stores. d. specialty retailers. e. warehouse showrooms.

d

As discount stores improve their services and atmosphere as well as raise their prices, the distinction between discount stores and ____ is becoming blurred. a. superstores b. supermarkets c. specialty stores d. department stores e. category killers

d

Direct selling, direct marketing, and vending machines are all examples of a. producing. b. advertising. c. promoting. d. retailing. e. wholesaling.

d

In contrast to industrial products, consumer products are often purchased because of a. economic planning. b. necessity. c. prior planning. d. social influences. e. rational needs.

d

What type of retailers are the primary competitors for specialty retailers? a. Discount stores b. Category killers c. Catalog marketers d. Department stores e. Superstores

d

What are outlet shopping centers?

stores carrying traditional manufacturer brands, such as Polo Ralph Lauren, Nike, and Guess.

Starbucks has an agreement with Pepsi Co. through which Pepsi distributes Starbucks' coffee drink, Frappucino, to grocery stores and other retail outlets. This is an example of a(n):

strategic channel alliance.

A franchise is an arrangement in which a supplier grants dealers the right to sell products in exchange for some type of consideration. a. True b. False

true

Coupons are a form of sales promotion. a. True b. False

true

Two ways to pull up higher in the organic search rankings is through appropriate use of key words and linking.

True

Warehouse clubs are able to offer a broad range of merchandise because their product lines are shallow and their sales volumes high. a. True b. False

True

Whereas the marketing channel is a group of interrelated organizations that direct products to customers, physical distribution deals with physical movement and inventory holding both within and among intermediaries

True

Wholesalers may provide marketing activities for those above and below them in the marketing channel.

True

You can "eliminate the middleman" but you can't eliminate the middleman's functions.

True

Terms for the Concept of Price

Tuition, Fee, premium, Retainer, Dues

Which of the following pricing approaches is used most often by retailers?

markup pricing.

The pricing strategy that assumes that demand is relatively inelastic over certain price ranges is called

price lining.

What equation shows organizations the relationship between price and profit?

(Price × Quantity Sold) Total Costs = Profits

In order to justify direct-response marketing, a product really needs to be priced above _____.

$20

Equation that shows the relationship between price and profit

(Price x Quantity Sold) - Total Costs = Profit

What are the three types of specialty retailers?

1. Traditional specialty retailers 2. Category Killers 3. Off-price retailes

What are the different types of direct marketing?

1. Catalog marketing 2. Direct-response marketing 3. Telemarketing 4. Television home shopping 5. Online retailing

What are the two broad categories of merchant wholesalers?

1. Full service 2. Limited service

What are the two categories of brick-and-mortar retail stores?

1. General merchandise retailers 2. Specialty retailers

What are the three types of full-service merchant wholesalers?

1. General-merchandise wholesalers 2. Limited-line wholesalers 3. Specialty-line wholesalers

What are the three general types of wholesaling establishments?

1. Merchant wholesalers 2. Agents and brokers 3. Manufacturers' sales branches and offices

What are the different types of shopping centers?

1. Neighborhood 2. Community 3. Regional 4. Superregional 5. Lifestyle 6. Power 7. Outlet

What are convenience stores?

A small self-service store that is open long hours and carries a narrow assortment of products, usually convenience items. Ex: Shell, Valero, 7-11

What is direct-response marketing?

A type of marketing in which a retailer advertises a product and makes it available through mail or telephone orders. Ex: television commercial offering exercise machines, cosmetics, or household cleaning products available through a toll-free number

What is catalog marketing?

A type of marketing in which an organization provides a catalog from which customers make selections and place orders by mail, telephone, or the internet

What are power shopping centers?

A type of shopping center that combines off-price stores with category killers

What are lifestyle shopping centers?

A type of shopping center that is typically open air and features upscale specialty stores, dining, and entertainment

Quantity discounts

Discounts offered for purchasing a large number of units.

Skimming pricing policy

Approach to pricing in which the seller charges a relatively high price on a new product initially in order to recover costs and make profits rapidly and then lowers the price at a later date to make sales to more price-sensitive buyers.

price elasticity of demand

Dividing the percentage change in quantity demanded by the percentage change in price gives the

What are commission merchants?

Agents that receive goods on consignment from local sellers and negotiate sales in large, central markets

After selecting a pricing strategy, what is the next step in the establishment of prices?

Answer: Determining a specific price​

Which of the following is true about the target market's evaluation of price?

Answer: The importance of price depends on the type of product.

Franchising

Agreement in which a supplier (franchiser) grants a dealer (franchisee) the right to sell products in exchange for some type of consideration. franchiser furnishes equipment, buildings, management, mkting franchisee supplies labor and capital, operation of business (subway, mcdonalds) advantages: enables startup, provides developed and proven business to franchisee, immediate market entry, success disadvantages: control over aspects of business,expensive of royalties and fees, lack of control of franchisees

Penetration pricing policy

Approach to pricing in which the seller charges a relatively low price on a new product initially in order to grow a market, gain market share, and discourage competition from entering the market.

Facilitating Agents

Are not channel members but provide limited services to channel members... Public Warehouses Finance Companies Ad Agencies Transportation Companies Trade Shows/Marts Factoring Houses

The Armadillo's Revenge is an off-beat steak house that features highway signs, parts of diesel tractor trucks, and billboards as part of the interior decor. Restaurant personal are dressed as paramedics and colorful armadillo posters decorate the ceiling. These surroundings are designed to attract a certain type of target customer. This is referred to as:

Atmospherics

When an organization sets a number of prices for selected groups of merchandise, this is commonly referred to as

price lining.

Amtrak prices its tickets so that it is less expensive to travel on weekends than during the week when there is heavy business travel. This illustrates ___ pricing

demand-based

To determine the breakeven point in units, divide the fixed costs by

price minus variable costs

What are general-merchandise wholesalers?

Full-service merchant wholesalers with a wide product mix but limited depth within product lines. They deal in products such as drugs, nonperishable foods, cosmetics, detergents, and tobacco.

What are specialty-line wholesalers?

Full-service wholesalers that carry only a single product line or a few items within a product line

What are rack jobbers?

Full-service, specialty-line wholesalers that own and maintain display racks in stores

The retailer is usually in an excellent position to:

Gain feedback from consumers.

The oldest form of exchange - trading of products - is known as

Barter

Sealed-bid pricing

Bidding process in which each seller submits a sealed bid and attempts to price below competition in order to get the contract. Many large construction and military projects are bid this way.

what differentiates from "for profit" marketing?

Bosses (board) don't oversee as directly— More creativity—fewer dollars/less supervision Sometimes controversial More diverse beneficiaries

Which of the following is not a common feature of a warehouse club?

Good customer service

Most Popular Search Engine

Google

Which of the following pricing objectives sets prices to recover cash as quickly as possible?

Cash flow

Which of the following types of retailing began with Montgomery Ward in the late 1800s?

Catalog marketing

Which of the following products would probably use the longest channel of distribution?

Chewing gum.

What are selling agents?

Intermediaries that market a whole product line or a manufacturer's entire output

Rate-of-return pricing

Cost-oriented approach to pricing that involves adding a desired rate of return on investment to total costs. Generally, a break even analysis is performed for expected production and sales levels and a rate of return is added on.

Who are merchant wholesalers?

Independently owned businesses that take title to goods, assume ownership risks, and buy and resell products to other wholesalers, business customers, or retailers

Unique Characteristics of services

Intangibility Inseparability of production & consumption Perishability Heterogeneity Client-based relationships Customer contact

What does the demand curve for a prestige product look like?

It forms a curve where the greatest quantity sold comes at a medium price and the quantities fall as the price increases or decreases.

Channel Conflict

Dual Distribution issues: Gray Marketers (diverters) Freeriders disagreements among marketing channel members on goals, roles, and rewards - who should do what and for what rewards

Many retailers now engage in multichannel retailing, which is what?

Employing multiple distribution channels that complement brick-and-mortar stores with websites, catalogs, and apps where consumers can research products, read other buyers' reviews, and make actual purchases. brick and mortar + online stuff

If a manufacturer uses UPS to deliver a rush package to a retailer, UPS is a\an:

Facilitating agent.

Sam's Widgets used to be available everywhere. But, as Sam became more successful, he specialized in more expensive widgets that not every household could afford. Accordingly, Sam reduced the number of Widget outlets to a few in each town and gave more Widget business to his dealers who proved they could support the product well. Sam is moving from which kind of distribution strategy to which other kind?

Intensive to selective.

If some merchants in the Battlefield Mall refuse to pay their mall association dues, this may lead to vertical channel conflict.

False

One reason that franchising is growing so rapidly is that franchise agreements only loosely describe the franchisor's and franchisee's respective rights and obligations, giving both considerable competitive flexibility in management decisions.

False

Retailers are frequently found in business product channels. a. True b. False

False

The shortest possible distribution channel would be producer to retailer to consumer. a. True b. False

False

When AT&T acquired Time Warner, this was an example of a contractual VMS. a. True b. False

False

Channel Integration

Horizontal Integration—Act of acquiring other operations at same level of channel ie., Safety buys Kroger AT&T buys MCI Vertical Integration-Combining two or more levels of marketing channels..Wal-mart buys a manufacturer or Ann Taylor develops retail outlets....

Competition-based pricing is

IS used when costs and revenues are secondary to competitors' prices.

If Wrigley set its pricing objective as attaining 38 percent of the chewing gum market, what else would be needed to make this a true pricing objective?

Identification of a time period for accomplishment

What are manufacturers' agents?

Independent intermediaries that represent two or more sellers and usually offer customers complete product lines

A manufacturer has decided to improve its inventory management by maintaining low inventory levels and waiting to purchase materials until right before they are needed in production. This inventory management technique is called:

Just-in-time (JIT)

Slotting allowances

Payments to retailer to get them to stock items on their shelves, a common tactic for getting new products into stores.

Which of the following retail stores invests the most in atmospherics?

Neiman Marcus

Sellers that emphasize distinctive product features to encourage brand preferences among customers are practicing

Nonprice competition

Dual Distribution

Operating through more than one channel I.e, Barnes and Noble with "Brick and Mortar" stores and Internet site.... Coke—Vending Machines, Corporate, Distributors

Which of the following would be used in setting price of a new product if considerable competition is expected? -psychological pricing -penetration pricing -odd-even pricing -price skimming -prestige pricing

Penetration pricing

A manager at JC Penney discovers that Sears has reduced the price of its children's Levi's from $31.99 to $24.99, according to an advertisement in the Sunday newspaper. She immediately phones her store and instructs the salesperson on duty to put a sign up next to their children's Levi's that reads, "SALE: $24.99." This is an example of what pricing strategy?

Random discounting

If a business decides to reduce its prices once in a while on an unsystematic basis, it is using

Random discounting

A Macy's manager designs the casual clothing department such that one of Macy's private label pairs of jeans, priced at $24.99, is positioned next to a national brand of jeans, such as Levis, priced at $39.99. What is the manager attempting to accomplish?

Reference pricing strategy

A Macy's manager designs the casual clothing department such that one of Macy's private label pairs of jeans, priced at $24.99, is positioned next to a national brand of jeans, such as Levis, priced at $39.99What is the manager attempting to accomplish?

Reference pricing strategy

48. When Sophie buys organic produce for her household using a channel with only one intermediary, that intermediary is classified as a a) retailer. b) wholesaler. c) broker. d) functional middleman. e) producer.

a) retailer.

Because of complaints that, depending on the bartender, the drinks being served in the Sea Breeze Hotel were either too weak or too strong, the manager installed a computerized bar that mixed all beverages. This was a response to problems associated with service:

Standardization/heterogeneity

Question What type of pricing objective would an organization use if it were in a favorable position and desired nothing more?

Status quo

Which type of pricing objective can reduce a firm's risk by helping to stabilize demand for its products?

Status quo

Price is a key element in the marketing mix because it relates directly to...

The generation of total revnue

Direct Selling

The marketing of products to ultimate consumers through face-to-face sales presentations at home (Scentsy) (MOST EXPENSIVE FORM OF SELLING) benefits: personal attention convenience of time and place of presentation-at home limitations: high costs make it most expensive form of selling negative consumer view of direct selling

Which of the following is a requirement for setting pricing objectives?

The objectives should be explicitly stated.

Non Store Retailing

The selling of products outside the confines of a retail facility (direct marketing) Catalog Marketing Direct response marketing Telemarketing Television home shopping Online retailing

Which of the following statements about markup pricing is correct?

The use of similar markups reduces price competition

Direct Marketing

The use of telephone, internet, and nonpersonal media to introduce products to consumers, who then can purchase them via email, telephone, or the internet.

What is direct marketing?

The use of the telephone, internet, and nonpersonal media to introduce products to customers, who can then purchase them via mail, telephone, or the internet

Price

The value that is exchanged for products in a marketing transaction

What do specialty retailers sell?

They offer substantial assortments in a few product lines. NOT specialty items

What is wholesaling?

Transactions in which products are bought for resale, for making other products, or for general business operations

Retailing

Transactions in which ultimate consumers are the buyers, when buyers intend to consume the product

155. Which mode of transportation hauls more freight than any other? a) Railroads b) Waterways c) Pipelines d) Trucks e) Airways

a) Railroads

Which mode of transportation hauls more freight than any other? a) Railroads b) Waterways c) Pipelines d) Trucks e) Airways

a) Railroads

A price-skimming strategy assumes that

all consumers have homogeneous tastes.

When a company prices one item in a line low with the intention of selling a higher-priced item in the same line, it is using

bait pricing

The Target bullseye is a classic example of a brand name. brand. brand mark. trade name. brand symbol.

brand mark

If Glade, which markets Glade scented Plug Ins and Glade scented candles and wax melts were to introduce Glade laundry detergent and dryer sheets with the same scents, this would be called ____ branding. a. individual b. family c. licensed d. brand-extension e. new-product

brand- extension

Jared is developing a business plan for a new type of bicycle helmet. He is interested in finding the point at which the costs of producing the helmet will equal the revenue earned from selling the product. Jared is interested in finding the

breakeven point

Chandra has extensive contacts in the real estate business and brings together buyers and sellers. Although she assumes no risks, she can offer specialized knowledge about real estate. Chandra is a _____.

broker

111. Order processing, inventory management, materials handling, warehousing, and transportation are the activities that define a) wholesaling. b) retailing. c) physical distribution. d) channel management. e) drop shipping.

c) physical distribution.

The pricing strategy that assumes that demand is relatively inelastic over certain price ranges is called

price lining

137. Eric is concerned about not having enough air conditioning units in inventory during June, July, and August so he keeps extra inventory to guard against stockouts during this critical period. Eric is keeping a) short order lead times. b) just-in-time inventory. c) a controlled usage rate. d) safety stock. e) excessive inventory.

d) safety stock.

When establishing prices, a marketer's first step is to:

develop pricing objectives

Freight transportation companies that offer several different shipment methods are called:

megacarriers.

Price skimming and penetration pricing are both strategies used for

new-product pricing

Telemarketing

performance of marketing-related activities by telephone

All of the following are psychological techniques except

price skimming.

Off-price retailers and category killers are both:

specialty retailers.

an external reference point

​Lucy buys a new dress at T.J. Maxx that has a price tag with "Compare at $150.00. Our Price $89.99." This is an example of the use of

interpretation

​What a price means or what it communicates to customers is called

What do category killers compete on the basis of?

1. low prices 2. product availability

Types of Distribution

Intensive—Widely distributed, convenience products Selective—Shopping products, limited distribution in selected market Exclusive—Specialty products, singular distribution in selected area

Direct selling is the most expensive form of retailing. a. True b. False

True

Which of the following is a common feature of a warehouse club? a. Annual fees b. Wide range of products c. Good customer service d. Moderate prices e. Pleasant atmosphere

a

Which of the following would be used in setting the price of a new product if considerable competition is expected? Answers: a. Penetration pricing b. Price skimming c. Psychological pricing d. Prestige pricing e. Odd-even pricing

a

Odd-even pricing is

a psychological printing strategy

A retailer engaged in direct marketing would probably select television home shopping over online retailing to sell a new kitchen device because of:

a superior ability to demonstrate the product.

A producer is not likely to receive _____ from an industrial distributor.

aggressive promotion of its brand

36. Having products available when the customer wants them is called a) place utility. b) time utility. c) availability. d) possession utility. e) chronavailability.

b) time utility.

Where do supply chains start? a) Raw materials b) Suppliers c) Customers d) Producers e) Retailers

c) Customers

Producers of convenience products such as soft drinks, toothpaste, and breath mints are most likely to use _____ distribution. a) selective b) extensive c) intensive d) exclusive e) demand-based

c) intensive

The manager at Best Buy puts a sign up next to a Pioneer audio system that reads, "Only $199.99! $60 less than Circuit City." This is an example of what type of pricing strategy?

comparison discounting.

Eric is concerned about not having enough air conditioning units in inventory during June, July, and August so he keeps extra inventory to guard against stockouts during this critical period. Eric is keeping a) short order lead times. b) just-in-time inventory. c) a controlled usage rate. d) safety stock. e) excessive inventory.

d) safety stock.

When channel members are linked by legal agreements that specify each member's rights and responsibilities, _______ exists. a) horizontal channel integration b) an administered VMS c) a corporate VMS d) a channel captain e) a contractual VMS

e) a contractual VMS

If Nabisco wants to quickly gain a large market share with its new line of reduced-fat snack crackers, it should use

penetration pricing.

What do all of the following have in common: tuition, fee, premium, retainer, dues?

they are all different concepts of price

If a company increased its price from $100 to $120 and the percentage change in quantity demanded was 40 percent, the price elasticity of demand for this product is

-2

Reasons for Growth in Services

-Increasing Affluence -More Leisure Time -More Female Participation in Workforce -Longer Life Expectancy -Greater Complexity of Products -Greater Complexity of Life -Greater Concern for Environment -Technology Growth

What are the four types of limited-service merchant wholesalers?

1. cash-and-carry wholesalers 2. truck wholesalers 3. drop shippers 4. mail-order wholesalers

What are the types of general-merchandise retailers?

1. department stores 2. discount stores 3. convenience stores 4. supermarkets 5. superstores 6. hypermarkets 7. warehouse clubs 8. warehouse showrooms

A retailer of Real Dry deodorant prices it at $2.00; it costs the retailer $1.40. What is the approximate markup as a percentage of selling price?

30 percent

Question A retailer of Real Dry deodorant prices it at $2.00; it costs the retailer $1.40. What is the markup as a percentage of selling price?

30%

J.C. Penney's pays $16.50 for a six-ounce bottle of cologne and sells it for $25.95. Its markup as a percentage of cost is approximately ___________ percent for this product.

57

J.C. Penny's pays $16.50 for a six-ounce bottle of cologne and sells it for $25.95. Its markup as a percentage of cost is approximately ___ percent for this product

57

Glenwood Pet Hospital is considering implementing a new pricing strategy for its veterinarian services. After reviewing the previous three years' revenue, Glenwood finds that most of its customers bring their pets in for the required annual vaccinations and then only if the animal is ill. Glenwood's objective is to generate more income per customer on an annual basis. The hospital has previously priced its services by charging a flat fee for the office visit, a fee for each vaccine, and a fee for each type of examination beyond the basic office visit. Most customers pay the flat office fee and a fee for a rabies vaccine. Glenwood is now considering a new plan where the pet owner would pay one fee that would cover an office visit, the required rabies vaccine, and additional vaccines that prevent heartworm, kennel-cough, and fleas. Glenwood hopes to encourage the pet owners to view their pet's health as part of a prevention program, rather than a one-time annual visit. Reference: Ref 20-2 Glenwood is considering a markup pricing basis, with the cost for office visit plus vaccines at $45. If Glenwood were to add a markup of 33.3 percent of the costs, its price would be

60

Channel of Distribution

A channel of distribution, or group of interrelated intermediaries (middlemen) who direct products to consumers.... any series of firms or individuals who participate in the flow of products from producer to final user or consumer Two major types of intermediaries: Merchants—take title to merchandise and resell it and Agents and Brokers who receive a fee or commission for expediting exchanges.

With respect to retailing, what is the "party plan"?

A consumer acts as a host and invites friends to view merchandise in a group setting, where a salesperson demonstrates the products.

Cost-plus pricing

A cost-oriented approach that involves totaling up the costs of producing a product or completing a project and then adding on a percentage or fixed profit amount. This approach is used when costs are difficult to estimate in advance such as military weapon development.

Markup pricing

A cost-oriented pricing approach that involves adding a percentage to the invoice price in order to determine the final selling price. For example, if a retailer used a 50% markup on a product that was bought from a wholesaler for $1, the selling price to the consumer would be $1.50.

Prestige pricing policy

A form of psychological pricing that involves charging a high price to create a signal that the product is exceptionally fine.

Bundle pricing

A form of psychological pricing that involves selling several products together at a single price in order to suggest a good value.

What is television home shopping?

A form of selling in which products are presented to television viewers, who can buy them by calling a toll-free number and paying with a credit card

In franchising, which of the following is not a benefit that a franchiser gains?

A franchiser gains more control over how franchisees operate establishments.

Television Home Shopping

A from of selling in which products are presented to television viewers, who can buy them by calling a toll free number and using a credit card. (p90x)

Price elasticity

A measure of consumers' price sensitivity which is estimated by dividing relative changes in the quantity sold by relative changes in price. If demand is elastic, a slight lowering of price will result in a relatively large increase in quantity demanded.

In which of the following situations is dual distribution likely to be determined illegal?

A producer uses company outlets to dominate independent retailers that carry its products.

General-Merchandise Retailers

A retail establishment that offers a variety of product lines

What is a general-merchandise retailer?

A retail establishment that offers a variety of product lines that are stocked in considerable depth.

What is category management?

A retail strategy of managing groups of similar, often substitutable products produced by different manufacturers

What are neighborhood shopping centers?

A type of shopping center usually consisting of several small convenience and specialty stores.

What are community shopping centers?

A type of shopping center with one or two department stores, some specialty stores, and convenience stores

What are regional shopping centers?

A type of shopping center with the largest department stores, widest product mixes, and deepest product lines. Ex: many shopping malls

What are superregional shopping centers?

A type of shopping center with the widest and deepest product mixes that attracts customers from many miles away and often has special attractions

What are agents and brokers?

A type of wholesaler that negotiate purchases and expedite sales but do not take title to products

What is a category killer?

A very large specialty store that concentrates on a major product category and competes on the basis of low prices and product availability. Ex: Home Depot, Lowe's, Staples, Office Depot, Petco, PetSmart, Best Buy

Characteristics of Digital Marketing

Addressability -ability to identify customers before they make purchase and when they return (cookies to personalize) Interactivity -Can interact with customers in "real time. (Community) Accessibility -Ease with which we can access information. (Google it) Connectivity -Social media, Review sites, crowdsourcing, reviews Control -Consumer controls—Permission based marketing

What is retailing?

All transactions in which the buyer intends to consume the product through personal, family, or household use

Virtual Sites/ Gaming

Allows new and different ways for marketers to connect with customers. Virtual technology for meetings/recruiting

Odd pricing

Also called odd-even pricing, a form of psychological pricing in which the prices are set at one or a few cents or dollars below a round number in order to create the perception that the price is low, for example 99 cents or $129 rather than $1 and $130.

What is franchising?

An arrangement in which a supplier (franchisor) grants a dealer (franchisee) the right to sell products in exchange for some type of consideration

Services Defined

An intangible product involving a deed, performance or an effort that cannot be physically possessed. Can be provided through human or mechanical efforts.

What is a retailer?

An organization that purchases products for the purpose of reselling them to ultimate consumers

Price fixing

An unfair business practice outlawed by the Sherman Antitrust Act and the Federal Trade Commission Act that involves competitors in a market colluding to set the final price of a product.

Which pricing objective de-emphasizes price and can lead to a climate of nonprice competition in an industry?

Answer Status quo

When Gabriella logs on to Dell's website, she sees a notebook model priced well below $1,000. As she continues through the site to view the other options, she realizes the first one she saw was the cheapest model available, but she of course wants more features. Dell is utilizing

Answer bait pricing.

When a company prices one item in a line low with the intention of selling a higher-priced item in the same line, it is using

Answer bait pricing.

Maintaining or increasing market share

Answer can be achieved even if industry sales are flat or decreasing.

Question When products in an industry are relatively homogeneous and price is a key purchase consideration,

Answer competition-based pricing becomes more important.

Pricing strategies and methods

Answer help direct and structure the selection of a final price.

Products such as light bulbs, canned soft drinks, and ice cream sandwiches are usually priced using

Answer multiple-unit pricing.

For customers, value is a function of the product's

Answer quality attributes.

Markup is measured either as a percentage of _____ or a percentage of _____.

Answer selling price; cost

A sale that advertised prices "up to 65 percent off" the original price uses

Answer tensile pricing.

Marketers at organizations engaged in nonprice competition

Answer: need competitive price information to make sure that their products are priced at approximately the same level as the prices of competing brands.

A marketer is most likely to set prices according to a cash-flow objective when a

Answer: quick return on investment is desired

Specialty Retailers

Category Killers- concentrates on major product (Barnes&Noble) Traditional Specialty Retailer- narrow line but deep (Victoria's Secret) Off-Price Retailers- buy manufactures off season/returns (Mashalls)

Which of the following would have the greatest width at the retail level?

Coca Cola soda

The manager at Best Buy puts a sign up next to a Pioneer audio system that reads, "Only $199.99! $60 less than Circuit City." This is an example of what type of pricing strategy?

Comparison Discounting

Suppose that Rayban is considering a new line of sunglasses that would be sold in major department stores. The new line would be positioned as a more distinctive brand than the typical glasses sold through department stores, and would be priced higher than other brands in the store, but a lower price line than the current Rayban lines that are sold through more selective stores. In determining the price for this sunglass line, Rayban wants to gather information about all brands sold in department stores and about customers' perceptions of those brands. Reference: Ref 20-1 Rayban's plan of gathering information about the other brands sold in department stores, including their prices, would most likely be used in a __________ basis for pricing.

Competition

If PepsiCo sets its 12-pack at $3.99 to match the price charged by Coca-Cola, Pepsi is using which pricing method?

Competition-based

Aspects of Digital Marketing

Consumer and Business to Business Ability to Share Information-Wikis, Blogs, Social Networking, You Tube, Websites themselves Rapid Response, Expanded Customer Service, Reduced Costs and Reduced Geographic Barriers

Which of the following was developed in 1927 to supply milk, eggs, and other products for customers replenishing their supplies?

Convenience stores

For custom-made equipment or commercial construction projects, which pricing method is most likely used?

Cost-plus

Steinway produces concert grand pianos, often using the custom materials and designs desired by a specific customer. The average price of these pianos runs about $50,000 depending on the exact piano. What type of pricing does Steinway most likely use for these pianos?

Cost-plus

The federal government often uses _____ pricing when it grants defense contracts.

Cost-plus

Major types of Retail Stores

Department Stores- large organizations retail stores (Macys) Discount Stores- self-service, brand name ant discount (Walmart) Supermarkets- large self-service, line of foods, nonfood (Kroger) Superstores- giant retail outlets that carry food/nonfood (SuperTargt) Hypermarkets- combine super market & discount in one (Carrefour) Warehouse clubs- Sam's club Warehouse showrooms - IKEA Convenience stores- small self service (7eleven)

Caruthers Paint Manufacturing Company buys the chemicals it needs for producing its products from a chemical producer, Roth Chemicals. In this instance, through which of the following types of channels are the chemicals being distributed to Caruthers?

Direct distribution

A channel of distribution is a group of individuals and organization that:

Directs the flow of products from producers to customers.

Which type of retailer generally accepts lower margins than traditional retailers in exchange for higher sales volume?

Discount stores

What are limited-line wholesalers?

Full-service wholesalers that carry only a few product lines but many products within those lines

Major Types of Retail Stores

General-Merchandise Retailers Specialty Retailers Category Killers

What are superstores?

Giant retail outlets that carry food and non-food products found in supermarkets, as well as most routinely purchased consumer products. Ex: Walmart Superstore

Which stage in the product life cycle is critical to a product's survival because competitive reactions to the product's success during this period will affect the product's life expectancy? a. Expansion b. Introduction c. Stabilization d. Decline e. Growth

Growth

If Wrigley set its pricing objective as attaining 38 percent of the chewing gum market, what else would be needed to make this a true pricing objective?

Identification of a time period for accomplishment

What is retail positioning?

Identifying an unserved or underserved market segment and serving it through a strategy that distinguishes the retailer from others in the minds of consumers in that segment.

Deceptive pricing

Illegal under the Federal Trade Commission Act, an approach that involves price deals that mislead the consumer. For example, putting a fake price on a product much higher than the product sells for in the market, crossing it out, and then offering the product at the market price and claiming a price reduction could easily mislead customers.

Paper towels and trash-can bags are most likely distributed through ____ and ____. a. exclusive distribution; a single channel b. multichannel distribution; convenience channels c. selective distribution; multiple channels d. intensive distribution; multichannel distribution e. strategic channel alliances; intensive distribution

Intensive distribution, multichannel distribution

What are department stores?

Large retail organizations characterized by a wide product mix and organized into separate departments to facilitate marketing efforts and internal management. Ex: Macy's, Nordstrom, Dillards

Category Killers

Large specialty store that concentrates on a major product category and competes on the basis of low prices and enormous product availability. They expand rapidly and gain sizable market shares, taking business away from smaller, high-cost retail outlets. PetSmart Home Depot Barnes and Noble Toys'R'Us Office Depot Best Buy

What are supermarkets?

Large, self-service stores that carry a complete line of food products, along with some non-food items. Ex: HEB, Kroger, Whole Foods

What are warehouse clubs?

Large-scale, members-only establishments that combine features of cash-and-carry wholesaling with discount retailing. Ex: Sam's Club

Vertical and horizontal integration/expansion are carefully scrutinized for legality by the federal government. The main concern here is for:

Lessening of competition.

What are mail-order wholesalers?

Limited-service wholesalers that sell products through catalogs. They enable buyers to choose and order particular catalog items for delivery through various mail carriers.

What are drop shippers?

Limited-service wholesalers that take title to goods and negotiate sales but never actually take possession of products. They forward orders from retailers, business buyers, or other wholesalers to manufacturers and arrange for carload shipments of items to be delivered directly from producers to these customers.

What are truck wholesalers?

Limited-service wholesalers that transport products directly to customers for inspection and selection.

What are cash-and-carry wholesalers?

Limited-service wholesalers whose customers pay cash and furnish transportation. They usually handle a limited line of products with a high turnover rate, such as groceries, building materials, and electrical or office supplies.

A firm establishes which of the following pricing objectives to maintain or increase its product's sales in relation to total industry sales

Market share

A firm establishes which of the following pricing objectives to maintain or increase its product's sales in relation to total industry sales?

Market share

Relationship of Marketing and IT

Marketing is the slick body of the car, but IT is the engine! IT department is your friend

Types of wholesalers

Merchant (buy and sell) (middleman- closer to manufacturer) -full service wholesalers -limited service Agents and brokers

Sherri McRae wants to establish a fashion wholesaling firm. Because of the risks involved in the rapid obsolescence of high-fashion items, Sherri does not want to establish an organization in which she has title to the garments. Which of the following types of wholesale establishments has she ruled out?

Merchant wholesaler

What are full-service merchant wholesalers?

Merchant wholesalers that perform the widest range of wholesaling functions

What are limited-service merchant wholesalers?

Merchant wholesalers that provide some services and specialize in a few functions

Social Media Trends

Mobile Marketing Viral Marketing Apps Facebook Twitter Pinterest YouTube Instagram Linked In

Price is considered to be the variable in the marketing mix that is

Most flexible

Mattie went shopping for a new pair of Levi jeans yesterday. She saw jeans at a department store priced from $45 to $59, but she chose not to buy any because she thought they were too expensive. On her way home she passed a stand where a man was selling Levi jeans for $20 a pair, and she bought two pairs from him. The sign in front of his stand announced that he had bought the jeans from a retailer who had gone out of business. Mattie's jeans were being sold:

On the gray market.

Product and target market characteristics usually determine the type of coverage a product receives. For which of the following products is selective distribution most appropriate?

Organic foods

Which of the following best describes wholesalers?

Organizations or individuals who buy products for resale to government, reseller, producer, and institutional users

Retailers

Organizations that purchase products for the purpose of reselling them to ultimate consumers (from wholesaler)

A concert is given for which 5,000 seats remain unsold. The impossibility of saving those seats for sale at another time is related to which characteristic of services?

Perishability

Predatory pricing

Practice that involves charging a very low price for a product with the intent of driving competitors out of business. It is illegal under the Sherman Act and Federal Trade Commission Act.

A product that has more features than those of its competition, or that is perceived to be of higher quality, warrants using which type of pricing strategy

Premium pricing

Breyer's produces a variety of ice cream flavors and lines of varying qualities. The higher quality ice cream varieties are priced higher than the basic ones. Breyer's is using _____ to price its ice cream.

Premium pricing

Breyer's produces a variety of ice cream flavors and lines of varying qualities. The higher quality ice cream varieties are priced higher than the basic ones. Breyer's is using ___ to price its ice cream

Premium pricing

selective or exclusive distribution

Premium-priced products are usually marketed through

When marketers emphasize price as an issue and match or beat the prices of other companies, they are using...

Price Competition

Promotional allowance

Price reduction offered to channel members in exchange for performing various promotional activities such as featuring the product in store advertising or on in-store displays.

Going-rate pricing

Pricing a product at the average charged in the industry.

Which of the following is least likely to be a factor affecting the selection of marketing channels?

Product packaging

What type of pricing strategy is used in a situation where demand for a product is price inelastic and the seller has an ethical responsibility not to overcharge the client?

Professional pricing

What type of pricing strategy is used in a situation where the seller has an ethical responsibility not to overcharge the client and the fees do not relate directly to the time and/or effort spent in specific cases?

Professional pricing

Which of the following pricing objectives is rarely operational because its achievement is difficult to measure?

Profit maximization

Push vs. Pull

Push: the idea that marketers are attempting to push their products at consumers. Common sales tactics include trying to sell merchandise directly to customers via company showrooms and negotiating with retailers to sell their products for them, or set up point-of-sale displays. Pull: to get the customers to come to you, hence the term pull, where marketers are attempting to pull customers in. Common sales tactics used for pull marketing include mass media promotions, word-of-mouth referrals and advertised sales promotions. From a business perspective, pull marketing attempts to create brand loyalty and keep customers coming back, whereas push marketing is more concerned with short-term sales.

A manager at JC Penney discovers that Sears has reduced the price of its children's Levi's from $31.99 to $24.99, according to an advertisement in the Sunday newspaper. She immediately phones her store and instructs the salesperson on duty to put a sign up next to their children's Levi's that reads, "SALE: $24.99." This is an example of what pricing strategy?

Random discounting

How google search works

Relvance, key words

Strategic Uses in Retailing

Retail Store Location Factors affecting location -Intended target market -kinds of products -suitability site for customer access -characteristic of existing retail operations Types of Location (How consumers perceive it, nordstrom -Freestanding structures -Traditional business districts Retail positioning -identifying an unserved market segment and serving it through a strategy that distinguishes the retailer from other in the minds of consumers in that segment. Store Image -Atmospherics -The physical element's in a store's design that appeal to consumers' emotions and encourage buying. -Interior layout, colors, furnishing, and lighting -Exterior storefront and entrance design, display windows and traffic congestion

What are warehouse showrooms?

Retail facilities in large, low-cost buildings with large on-premises inventories and minimal services. Ex: IKEA

Alex is preparing a new strategic plan for a large retailer. Several aspects of the retailer are to be evaluated in order to make plans and set goals. Of all the issues that Alex can select in developing the new strategic plan, which of the following is the least flexible?

Retail store location

Category Management

Retail strategy of managing groups of similar products that may be produced by different manufacturers. (determining space for products such as cosmetics, cereals)`

Any organization that purchases products for the purpose of reselling them to ultimate consumers is a(n)

Retailer

What are extreme-value stores?

Retailers that are a fraction of the size of conventional discount stores and typically offer very low prices on smaller size name-brand nonperishable household items. Ex: Dollar General, Family Dollar

Online Retailing

Retailing that makes products available to buyers through computer connections. (Amazon)

Which of the following prohibits price discrimination that lessens competition among wholesalers and retailers?

Robinson-Patman Act

Ways Marketers use the Web to Market

SEO/SEM - Search Engine Optimization (Free) PPC - Pay per click Emails/Eblasts Affiliate Marketing Social Networking/Branded Content Website Itself Apps

Experience and Credence Qualities of Service

Search Qualities-tangible attributes Experience Qualities-can be assessed only during/ after consumption Credence Qualities-customer may be unable to evaluate even after consumption[

Ethical/Legal Issues

Security Children and the Internet (COPA) Privacy Spam-unsolicited E-mail Online Fraud Intellectual Property/File sharing

What are discount stores?

Self-service, general-merchandise stores that offer brand-name and private-brand products at low prices. Ex: Target & Walmart

Services provided by wholesalers

Serve as an extension of the producer's sales force through contact with suppliers and retailers lend financial assistance for the distribution channel (pay for transportation goods, reduce producers warehousing expense, purchase producers entire output, channel info from sellers to buyers)

Value pricing

Setting prices so that targeted customers will perceive products to offer greater value than competitive offerings. For existing products, this can be accomplished by offering more product or service while maintaining or decreasing the dollar price.

What are off-price retailers?

Stores that buy manufacturers' seconds, overruns, returns, and off-season merchandise for resale to consumers at deep discounts. Ex: T.J. Maxx, Marshalls

What are traditional specialty retailers?

Stores that carry a narrow product mix with deep product lines. Ex: stores that sell apparel, jewelry, sporting goods, fabrics, computers, and pet supplies. Ex: Foot Locker, Sunglass Hut, and Kay's Jewelry are all traditional specialty retailers

What are hypermarkets?

Stores that combine supermarket and discount store shopping in one location

E-Marketing

Strategic process of creating, distributing, promoting and pricing products for targeted customers in the virtual environment of the Internet and beyond.

All of the following are reasons a company would likely emphasize advertising in its promotion mix except a. The company's customers are dispersed across a vast region b. The size of the product's target market consists of millions of consumers c. The company's products are seasonal in nature d. The company's promotion budget is extensive e. The company has chosen exclusive distribution for its products

The company has chosen exclusive distribution for its product

What is telemarketing?

The performance of marketing-related activities by telephone

What is asmospherics?

The physical elements in a store's design that appeal to consumers' emotions and encourage buying

Price discrimination

The practice of charging different prices to different buyers for goods of like grade and quality which is illegal under the Robinson-Patman Act if it lessens or is deemed injurious to competition.

product quality

The pricing of Clinique makeup considerably higher than brands such as Cover Girl, Revlon, and Maybelline is used to communicate ____, which is the company's primary pricing objective.

A brand name should indicate the product's major benefits. a. True b. False

True

A product need not be a physical product. a. True b. False

True

According to class lecture/notes, 70% of customers research a product online regardless of where they make their final purchase.

True

Approximately 80% of people will not look past the first page of a search. If they don't find what they are looking for in the first few items that pull up, they will tend to refine their search.

True

Co-branding capitalizes on the trust that customers have in both companies involved, especially when the brands complement one another in the mind of the buyer. a. True b. False

True

Compared to an unfamiliar brand, a familiar brand is more likely to be selected by a customer because it often is viewed as reliable and of acceptable quality. a. True b. False

True

Customers use brands to help judge product quality. T/F

True

Long-term commitments usually characterize the relationships among channel members. a. True b. False

True

Marketing intermediaries can reduce the cost of exchanges. a. True b. False

True

Merchant wholesalers assume risks of ownership.

True

Packaging can be used to communicate symbolically the quality or premium nature of a product. a. True b. False

True

The Web is sometimes referred to as a pull medium because customers determine which Web sites they visit.

True

The atmosphere and décor of a retail store, the variety and depth of product choices, the customer support, even the sounds and smells all contribute to the experiential element of its total product. a. True b. False

True

The first adopters of a product are the innovators. a. True b. False

True

Catalog Marketing

Type of marketing in which an organization provides a catalog from which customers can make selections and place orders by mail, telephone or the internet. Consumer advantages are efficiency and convenience MK advantages are lower location, facility selling costs Disadvantages inflexibility and limited selection and local service

Direct Response Marketing

Type of marketing that occurs when a retailer advertises a product and makes it available through mail or telephone orders. p90x

Digital Marketing

Uses all digital media to develop communication and exchanges with customers.

When consumers are making do with less expensive products and shopping more selectively, manufacturers and retailers must focus on the ___________ of their products. Answer price

VALUE

If a wine maker decides to buy a bottle labeling operation, it would be an example of:

Vertical integration

Structure of Channels

Vertical—Length of Channel Short channels, more control/contact with customers Long channels Horizontal—Number of participants of the same type at a given level of channel. i.e, Toy Industry—Manufacturers—Tyco, Mattel, Hasbro, Ty Retailers-Toys R Us, Wal-mart, Kaybee

Which of the following physical distribution functions involves design and operation of facilities for storing goods?

Warehousing

Nonprofit Marketing

Whereas the chief beneficiary of a business enterprise is whoever owns it or has stock in it, in theory, the only beneficiaries of a nonprofit organization are its clients, its member or the public at large.

Which of the following is true about wholesaling?

Wholesaling activities must be performed during distribution of all goods, whether or not a wholesaling institution is involved.

Which of the following is most likely to be a product stocked solely by an industrial distributor?

Wind turbines

The salvage value of unsold services is:

Zero.

By locating in the same general vicinity as other car dealerships, Hartfield Honda can a. facilitate comparison shopping. b. create form utility. c. facilitate wholesale exchanges. d. create possession utility. e. create exchange utility.

a

Colin brags to his friend Reid that his new bike shop has lots of parking and great visibility from Harper Avenue, and his nearest business neighbors are several hundred feet away. Colin has selected a ____________ site for his retail business. a. free-standing b. traditional business district c. neighborhood shopping center d. community shopping center e. regional shopping center

a

Compared with other types of retailers, department stores compete mainly on the basis of a. customer services. b. low prices. c. a very deep assortment of a few specific products. d. high sales volume. e. catalog sales.

a

Off-price retailers feature a. deep discounts, few customer services, and central checkouts. b. sales expertise, wide selections, and low prices. c. overstocks, designer brands, and moderate prices. d. a narrow product range and enormous product availability. e. cash only sales and few customer services.

a

Self-service, general merchandise stores such as Kmart, Walmart, and Target are known as a. discount stores. b. warehouse showrooms. c. catalog showrooms. d. superstores. e. specialty retailers.

a

The type of retail location that is commonly being preserved and revitalized in many cities is the a. traditional business district. b. free-standing structure. c. community shopping center. d. neighborhood shopping center. e. nontraditional shopping center.

a

Which of the following characteristics is unique to pricing more so than other variables of the marketing mix? Answers: a. Flexibility b. Consistency c. Challenging d. Truthfulness e. Clarity

a

Which type of retail outlet can have up to 200,000 square feet? a. Superstores b. Supermarkets c. Discount stores d. Department stores e. Category killers

a

Who provides the environment in which exchanges with ultimate consumers occur? a. Retailers b. Wholesalers c. Catalogers d. Telemarketers e. Producers

a

When channel members are linked by legal agreements that specify each member's rights and responsibilities, _____ exists.

a contractual VMS

One advantage of nonprice competition is that

a firm can build customer loyalty

Glenwood Pet Hospital is considering implementing a new pricing strategy for its veterinarian services. After reviewing the previous three years' revenue, Glenwood finds that most of its customers bring their pets in for the required annual vaccinations and then only if the animal is ill. Glenwood's objective is to generate more income per customer on an annual basis. The hospital has previously priced its services by charging a flat fee for the office visit, a fee for each vaccine, and a fee for each type of examination beyond the basic office visit. Most customers pay the flat office fee and a fee for a rabies vaccine. Glenwood is now considering a new plan where the pet owner would pay one fee that would cover an office visit, the required rabies vaccine, and additional vaccines that prevent heartworm, kennel-cough, and fleas. Glenwood hopes to encourage the pet owners to view their pet's health as part of a prevention program, rather than a one-time annual visit. Reference: Ref 20-2 Glenwood's previous pricing strategy is an example of __________ pricing, while the new strategy is an example of __________ pricing

a la carte; bundle

If Kroger Food Stores advertises 2-liter bottles of Pepsi for $0.89 to generate store traffic that will purchase other items at regular prices, the grocer is using

a price leader

If Kroger Food Stores advertises 2-liter bottles of Pepsi for 89 cents to generate store traffic that will purchase other items at regular prices, the grocer is using

a price leader.

Odd-even pricing is

a psychological pricing strategy.

A general merchandise retailer offers:

a variety of product lines that are stocked in depth.

When a single channel member manages an integrated marketing channel to achieve low-cost, efficient distribution for satisfying target markets, _____ exists.

a vertical marketing system

What is a primary difference between an industrial distributor and a manufacturers' agent? a) A manufacturers' agent does not acquire title nor usually take possession of the products whereas an industrial distributor does. b) A manufacturers' agent is employed by the manufacturers while an industrial distributor is independent. c) An industrial distributor is employed by the manufacturers while a manufacturers' agent is independent. d) A manufacturers' agent rarely adds any value to the marketing channel while an industrial distributor reduces costs significantly. e) An industrial distributor does not form relationships with customers for repeat business whereas a key asset of a manufacturers' agent is his knowledge of his customers.

a) A manufacturers' agent does not acquire title nor usually take possession of the products whereas an industrial distributor does.

175. If Anheuser-Busch refused to allow Jackson Distributing to carry its product if the distributor also carried Miller Brewing products, the company would have been engaging in which of the following channel management practices? a) Exclusive dealing b) Dual distribution c) Tying contacts d) Refusal to deal e) Restricted sales territories

a) Exclusive dealing

If Anheuser-Busch refused to allow Jackson Distributing to carry its product if the distributor also carried Miller Brewing products, the company would have been engaging in which of the following channel management practices? a) Exclusive dealing b) Dual distribution c) Tying contacts d) Refusal to deal e) Restricted sales territories

a) Exclusive dealing

164. Which of the following organizations consolidates shipments from several firms into more efficient lot sizes? a) Freight forwarder b) Forwarding agency c) Transporter d) Special transportation company e) Intermodal shipping company

a) Freight forwarder

Which of the following organizations consolidates shipments from several firms into more efficient lot sizes? a) Freight forwarder b) Forwarding agency c) Transporter d) Special transportation company e) Intermodal shipping company

a) Freight forwarder

161. ___________ is the combining and coordinating of two or more modes of transportation to take advantage of benefits offered by each of the different types of carriers. a) Intermodal transportation b) Physical distribution c) Containerized movement d) Efficient transportation e) Freight forwarding

a) Intermodal transportation

___________ is the combining and coordinating of two or more modes of transportation to take advantage of benefits offered by each of the different types of carriers. a) Intermodal transportation b) Physical distribution c) Containerized movement d) Efficient transportation e) Freight forwarding

a) Intermodal transportation

32. What links producers to consumers through the purchase and reselling of products or contractual agreements? a) Marketing intermediaries b) Distributors c) Suppliers d) Middle marketers e) Marketing channels

a) Marketing intermediaries

What links producers to consumers through the purchase and reselling of products or contractual agreements? a) Marketing intermediaries b) Distributors c) Suppliers d) Middle marketers e) Marketing channels

a) Marketing intermediaries

154. Which major mode of freight transportation provides the most flexible schedules and routes? a) Trucks b) Waterways c) Airways d) Pipelines e) Railroads

a) Trucks

Which major mode of freight transportation provides the most flexible schedules and routes? a) Trucks b) Waterways c) Airways d) Pipelines e) Railroads

a) Trucks

70. Starbucks has an agreement with Pepsi Co. through which Pepsi distributes Starbucks' coffee drink, Frappucino, to grocery stores and other retail outlets. This is an example of a) a strategic channel alliance. b) exclusive distribution. c) dual distribution. d) horizontal channel integration. e) channel leadership.

a) a strategic channel alliance.

Starbucks has an agreement with Pepsi Co. through which Pepsi distributes Starbucks' coffee drink, Frappucino, to grocery stores and other retail outlets. This is an example of a) a strategic channel alliance. b) exclusive distribution. c) dual distribution. d) horizontal channel integration. e) channel leadership.

a) a strategic channel alliance.

98. When a single channel member manages an integrated marketing channel to achieve low-cost, efficient distribution for satisfying target markets, ______ exists. a) a vertical marketing system b) horizontal channel integration c) channel power d) channel cooperation e) extensive distribution

a) a vertical marketing system

When a single channel member manages an integrated marketing channel to achieve low-cost, efficient distribution for satisfying target markets, ______ exists. a) a vertical marketing system b) horizontal channel integration c) channel power d) channel cooperation e) extensive distribution

a) a vertical marketing system

105. In an administered vertical marketing system (VMS), interorganizational relationships are a) achieved by informal coordination. b) formalized through contracts. c) combined under the ownership of a single organization. d) guided by legal agreements. e) achieved by clearly defining the obligations and rights of all channel members.

a) achieved by informal coordination.

In an administered vertical marketing system (VMS), interorganizational relationships are a) achieved by informal coordination. b) formalized through contracts. c) combined under the ownership of a single organization. d) guided by legal agreements. e) achieved by clearly defining the obligations and rights of all channel members.

a) achieved by informal coordination.

150. A public warehouse is a a) business that leases storage space and related facilities for distribution to other firms. b) large, centralized warehouse that focuses on moving rather than storing goods. c) company that provides a complete array of logistical services for businesses. d) company-operated facility for storing and shipping products. e) warehouse used to store the personal property of many different customers.

a) business that leases storage space and related facilities for distribution to other firms.

A public warehouse is a a) business that leases storage space and related facilities for distribution to other firms. b) large, centralized warehouse that focuses on moving rather than storing goods. c) company that provides a complete array of logistical services for businesses. d) company-operated facility for storing and shipping products. e) warehouse used to store the personal property of many different customers.

a) business that leases storage space and related facilities for distribution to other firms.

93. Marketing channel members are likely to experience misunderstandings, frustration, and poorly coordinated strategies as a result of a) channel conflict caused by inefficient communication between channel members. b) open communication among the channel members. c) methods of channel coordination designed to reduce ambiguity. d) negotiating territorial issues among regional distributors of a product. e) allowing one member of the channel to take the role of channel captain.

a) channel conflict caused by inefficient communication between channel members.

Marketing channel members are likely to experience misunderstandings, frustration, and poorly coordinated strategies as a result of a) channel conflict caused by inefficient communication between channel members. b) open communication among the channel members. c) methods of channel coordination designed to reduce ambiguity. d) negotiating territorial issues among regional distributors of a product. e) allowing one member of the channel to take the role of channel captain.

a) channel conflict caused by inefficient communication between channel members.

94. When produce companies such as Dole Tomatoes bypass wholesalers and sell directly to retailers, it is likely to create channel ___________ between Dole and these wholesalers. a) conflict b) cooperation c) leadership d) integration e) flows

a) conflict

When produce companies such as Dole Tomatoes bypass wholesalers and sell directly to retailers, it is likely to create channel ___________ between Dole and these wholesalers. a) conflict b) cooperation c) leadership d) integration e) flows

a) conflict

107. In the distribution channel for Domino's Pizza, channel members are linked by legal agreements that spell out the obligations and rights of each member. This is an example of a(n) ___________ vertical marketing system. a) contractual b) administered c) corporate d) negotiated e) institutional

a) contractual

In the distribution channel for Domino's Pizza, channel members are linked by legal agreements that spell out the obligations and rights of each member. This is an example of a(n) ___________ vertical marketing system. a) contractual b) administered c) corporate d) negotiated e) institutional

a) contractual

122. The United States Postal Service works hard to get priority mail from the sender to the recipient as quickly as possible in order to compete with companies such as UPS and FedEx. The postal service works to reduce a) cycle time. b) order processing. c) turn-around. d) shipping time. e) transportation.

a) cycle time.

The United States Postal Service works hard to get priority mail from the sender to the recipient as quickly as possible in order to compete with companies such as UPS and FedEx. The postal service works to reduce a) cycle time. b) order processing. c) turn-around. d) shipping time. e) transportation.

a) cycle time.

67. Del Monte markets ketchup for household use to supermarkets through grocery wholesalers. It markets ketchup for institutional use through industrial distributors and food brokers. Del Monte is using a) dual distribution. b) industrial distribution. c) strategic channel alliance. d) supply chain management. e) an unethical marketing channel.

a) dual distribution.

69. Gateway Computer makes computers available through its own stores, its toll-free telephone line, and a website. This is an example of a) dual distribution. b) vertical integration. c) horizontal integration. d) tying agreements. e) exclusive dealing.

a) dual distribution.

Del Monte markets ketchup for household use to supermarkets through grocery wholesalers. It markets ketchup for institutional use through industrial distributors and food brokers. Del Monte is using a) dual distribution. b) industrial distribution. c) strategic channel alliance. d) supply chain management. e) an unethical marketing channel.

a) dual distribution.

Gateway Computer makes computers available through its own stores, its toll-free telephone line, and a website. This is an example of a) dual distribution. b) vertical integration. c) horizontal integration. d) tying agreements. e) exclusive dealing.

a) dual distribution.

57. An independent business that takes title to products and carries inventories is a(n) a) industrial distributor. b) intermediary. c) agency. d) wholesaler. e) producer.

a) industrial distributor.

An independent business that takes title to products and carries inventories is a(n) a) industrial distributor. b) intermediary. c) agency. d) wholesaler. e) producer.

a) industrial distributor.

136. A manufacturer has decided to improve its inventory management by maintaining low inventory levels and waiting to purchase materials until right before they are needed in production. This inventory management technique is called a) just-in-time (JIT). b) time management. c) inventory minimization. d) economic order quantity. e) reorder point maximization.

a) just-in-time (JIT).

A manufacturer has decided to improve its inventory management by maintaining low inventory levels and waiting to purchase materials until right before they are needed in production. This inventory management technique is called a) just-in-time (JIT). b) time management. c) inventory minimization. d) economic order quantity. e) reorder point maximization.

a) just-in-time (JIT).

35. If Nokia decides to make changes in its marketing channels, the strategic significance is that channel decisions are a) long-term commitments. b) short-term commitments. c) easier to change than prices. d) easier to change than promotion. e) impossible to change.

a) long-term commitments.

If Nokia decides to make changes in its marketing channels, the strategic significance is that channel decisions are a) long-term commitments. b) short-term commitments. c) easier to change than prices. d) easier to change than promotion. e) impossible to change.

a) long-term commitments.

165. Freight transportation companies that offer several different shipment methods are called a) megacarriers. b) intermodal transporters. c) freight forwarders. d) shipping experts. e) superfreighters.

a) megacarriers.

Freight transportation companies that offer several different shipment methods are called a) megacarriers. b) intermodal transporters. c) freight forwarders. d) shipping experts. e) superfreighters.

a) megacarriers.

114. If a firm decides to contract its physical distribution functions to third parties that have no managerial authority within the marketing channel, it is using a) outsourcing. b) sole-sourcing. c) logistics. d) wholesaling. e) distribution services.

a) outsourcing.

If a firm decides to contract its physical distribution functions to third parties that have no managerial authority within the marketing channel, it is using a) outsourcing. b) sole-sourcing. c) logistics. d) wholesaling. e) distribution services.

a) outsourcing.

37. Marketing channels create three types of utility for consumers including a) place, time, and possession. b) location, availability, and suitability. c) time, location, and promotion. d) retailer, wholesaler, and producer. e) position, possession, and place.

a) place, time, and possession.

Marketing channels create three types of utility for consumers including a) place, time, and possession. b) location, availability, and suitability. c) time, location, and promotion. d) retailer, wholesaler, and producer. e) position, possession, and place.

a) place, time, and possession.

54. Steelcase, Inc. markets furniture directly to businesses. This is an example of a(n) _________ channel. a) producer-to-business buyer b) producer-to-industrial-distributor-to-business buyer c) producer-to-agent-to-business buyer d) equipment e) consumer

a) producer-to-business buyer

Steelcase, Inc. markets furniture directly to businesses. This is an example of a(n) _________ channel. a) producer-to-business buyer b) producer-to-industrial-distributor-to-business buyer c) producer-to-agent-to-business buyer d) equipment e) consumer

a) producer-to-business buyer

47. After direct-marketing, the next slightly longer marketing channel adds a(n) a) retailer. b) producer. c) wholesaler. d) agent. e) consumer.

a) retailer.

After direct-marketing, the next slightly longer marketing channel adds a(n) a) retailer. b) producer. c) wholesaler. d) agent. e) consumer.

a) retailer.

When Sophie buys organic produce for her household using a channel with only one intermediary, that intermediary is classified as a a) retailer. b) wholesaler. c) broker. d) functional middleman. e) producer.

a) retailer.

80. Using only some of the available outlets to distribute a product is called a) selective distribution. b) intensive distribution. c) channel conflict. d) vertical channel integration. e) exclusive distribution.

a) selective distribution.

Using only some of the available outlets to distribute a product is called a) selective distribution. b) intensive distribution. c) channel conflict. d) vertical channel integration. e) exclusive distribution.

a) selective distribution.

23. Long-term partnerships among channel members working together to reduce inefficiencies, costs, and redundancies in the entire marketing channel is called a) supply chain management. b) vertical channel integration. c) industrial management. d) industrial distribution. e) marketing management.

a) supply chain management.

28. Wal-Mart is working with its suppliers, using tools such as electronic billing, purchase order verification, and bar code technology, to integrate data used to improve overall performance. This is an example of a) supply chain management. b) a vertical marketing system. c) a horizontal marketing system. d) channel conflict. e) dual distribution.

a) supply chain management.

Long-term partnerships among channel members working together to reduce inefficiencies, costs, and redundancies in the entire marketing channel is called a) supply chain management. b) vertical channel integration. c) industrial management. d) industrial distribution. e) marketing management.

a) supply chain management.

Wal-Mart is working with its suppliers, using tools such as electronic billing, purchase order verification, and bar code technology, to integrate data used to improve overall performance. This is an example of a) supply chain management. b) a vertical marketing system. c) a horizontal marketing system. d) channel conflict. e) dual distribution.

a) supply chain management.

124. Order processing is defined as a) the receipt and transmission of sales order information. b) the second stage in a physical distribution system. c) the four main tasks. d) the same as order handling. e) electronic processing of information.

a) the receipt and transmission of sales order information.

Order processing is defined as a) the receipt and transmission of sales order information. b) the second stage in a physical distribution system. c) the four main tasks. d) the same as order handling. e) electronic processing of information.

a) the receipt and transmission of sales order information.

170. The main reason a manufacturer will prohibit intermediaries from selling its products outside designated sales territories is to a) tighten its control over distribution of its products. b) discourage competition from other manufacturers. c) incorporate selective distribution. d) contain distribution costs. e) punish intermediaries for past behavior.

a) tighten its control over distribution of its products.

The main reason a manufacturer will prohibit intermediaries from selling its products outside designated sales territories is to a) tighten its control over distribution of its products. b) discourage competition from other manufacturers. c) incorporate selective distribution. d) contain distribution costs. e) punish intermediaries for past behavior.

a) tighten its control over distribution of its products.

141. Johnson Controls hires a consultant to assess its materials handling procedures. The consultant recommends the implementation of ___________ if they want easier movement of items between internal destinations in the warehouse. a) unit loading b) piggybacking c) containerization d) inventory management e) priority transportation

a) unit loading

Johnson Controls hires a consultant to assess its materials handling procedures. The consultant recommends the implementation of ___________ if they want easier movement of items between internal destinations in the warehouse. a) unit loading b) piggybacking c) containerization d) inventory management e) priority transportation

a) unit loading

118. The best way to reduce overall distribution costs is to a) use a total-cost approach to analyze and evaluate the entire system. b) minimize the costs associated with materials handling and transportation. c) reduce the number of distribution functions that are necessary. d) find the cheapest price for each function and use them to create the system. e) lower the service standards that the company has.

a) use a total-cost approach to analyze and evaluate the entire system.

The best way to reduce overall distribution costs is to a) use a total-cost approach to analyze and evaluate the entire system. b) minimize the costs associated with materials handling and transportation. c) reduce the number of distribution functions that are necessary. d) find the cheapest price for each function and use them to create the system. e) lower the service standards that the company has.

a) use a total-cost approach to analyze and evaluate the entire system.

An arrangement where a producer forbids an intermediary to carry products made by competing manufacturers is called Select one: a. exclusive dealing. b. exclusive distribution. c. refusal to deal. d. a tying agreement. e. contractual VMS.

a.

Channel decisions are important to marketers mostly because Select one: a. they involve long-term commitments and affect customer accessibility. b. consumers value reasonable prices delivered through marketing channels. c. many businesses are marketing intermediaries. d. they are relatively flexible to change quickly. e. they dictate what promotional strategies companies should use.

a.

Eliminating a wholesaler from a marketing channel will Select one: a. not eliminate the functions performed by that wholesaler. b. eliminate the functions performed by that wholesaler. c. lead to lower costs but higher prices. d. reduce channel conflict. e. cut costs and lower prices.

a.

Goodyear allows companies like Sears and Discount Tire to distribute and discount its tires. This action significantly increases the possibility of channel ________ with independent Goodyear dealers. Select one: a. conflict b. power c. leadership d. understanding e. communication

a.

Which of the following business marketing channels is most likely to be used? a. producer to customer b. producer to agent to industrial distributor to customer c. producer to manufacturer's agent to customer d. producer to industrial distributor to customer e. producer to industrial distributor to retailer to customer

a. producer to consumer

Which of the following is often favored by intermediaries? a. restricted sales territories b. refusal to deal c. full-line forcing d. exclusive dealing e. tying agreements

a. restricted sales territories

rob stevens is the head of a company that produce computer software for production scheduling. the firm is small and presently does not generate enough volume to justify hiring a sales force. the firm is probably using ___ to maintain contact with the firms using its products: -wholesalers -brokers -agents -merchants -retailers

agents

Which of the following products is most likely to have an elastic demand curve?

airline tickets for vacation travel

A store that offers a wide variety of shoes for men, women, and children would most likely be considered a(n) a. department store. b. specialty retailer. c. category killer. d. off-price retailer. e. warehouse showroom.

b

After shopping in the same store for nearly two hours, Chelsea goes to Burger King for a Whopper, while Cynthia goes to the optical shop to see if her contacts are ready. They engage in these activities without leaving the store in which they have been shopping. They are most likely in a a. superstore. b. hypermarket. c. department store. d. general merchandise retailer. e. discount store.

b

Category killers compete primarily on the basis of a. enormous product selection and sales expertise. b. low prices and enormous product availability. c. convenient locations and customer services. d. rock-bottom prices and moderate selections. e. one-stop shopping and product availability.

b

If a store has areas for men's apparel, women's apparel, housewares, cosmetics, and jewelry and competes mostly on the basis of service, it is a(n) a. off-price retailer. b. department store. c. discount store. d. superstore. e. supermarket.

b

Maria recently put her house on the market at an asking price of $260,000. She realizes, however, that in order to sell the house, she may have to use Answers: a. price lining. b. negotiated pricing. c. professional pricing. d. secondary-market pricing. e. reference pricing.

b

Ollies' is a regional self-service retail store that sells name-brand electronics, tools, housewares, and sporting goods at low prices. Ollies' is an example of a a. department store. b. discount store. c. warehouse showroom. d. specialty store. e. superstore.

b

Pricing the basic product in a product line low while pricing related items at a higher level is called Answers: a. premium pricing. b. captive pricing. c. price lining. d. price skimming. e. bait pricing.

b

Retailing is best characterized as a. large organizations that carry wide and deep product mixes. b. transactions in which the buyer intends to consume the product through personal, family, or household use. c. arrangements whereby a supplier grants a dealer the right to sell its products. d. transactions in which the purchaser intends to use the product for resale or for business operations. e. exchanges that take place only in a store or service establishment.

b

Stores such as T.J. Maxx, Burlington Coat Factory, and Marshalls buy manufacturers seconds, overruns, returns, and off-season production runs at below wholesale prices. These firms resell this merchandise to consumers at deep discounts, and are called a. category killers. b. off-price retailers. c. specialty retailers. d. hypermarkets. e. discount retailers.

b

Supermarkets, discount stores, and hypermarkets can all be classified as a. superstores. b. general merchandise retailers. c. department stores. d. specialty retailers. e. off-price retailers.

b

The kinds of products being sold and the availability of public transportation are both factors that influence a retailer's decision about a. store atmosphere. b. location. c. retail positioning. d. store image. e. the wheel of retailing.

b

157. If a microchip in a computer in New York failed, which method of transportation would be most appropriate to use to replace the chip immediately if one could get the new chip only from Silicon Valley, California? a) Pipeline b) Airways c) Waterways d) Truck e) Railroad

b) Airways

If a microchip in a computer in New York failed, which method of transportation would be most appropriate to use to replace the chip immediately if one could get the new chip only from Silicon Valley, California? a) Pipeline b) Airways c) Waterways d) Truck e) Railroad

b) Airways

56. Caruthers Paint Manufacturing Company buys the chemicals it needs for producing its products from a chemical producer, Roth Chemicals. In this instance, the chemicals are being distributed to Caruthers through which of the following types of channels? a) Industrial distributor b) Direct distribution c) Retail d) Wholesaler-sponsored e) Producer

b) Direct distribution

Caruthers Paint Manufacturing Company buys the chemicals it needs for producing its products from a chemical producer, Roth Chemicals. In this instance, the chemicals are being distributed to Caruthers through which of the following types of channels? a) Industrial distributor b) Direct distribution c) Retail d) Wholesaler-sponsored e) Producer

b) Direct distribution

52. Manufacturers of convenience products such as chewing gum reach customers through thousands of retailers. What marketing channel are these manufacturers most likely to use? a) Producer, consumer b) Producer, wholesaler, retailer, consumer c) Producer, wholesaler, agent, retailer, consumer d) Producer, retailer, consumer e) Retailer, consumer

b) Producer, wholesaler, retailer, consumer

85. For which of the following products would exclusive distribution be most appropriate? a) Gasoline b) Rolls Royce automobile c) Washing machine d) Laundry detergent e) Moderately priced luggage

b) Rolls Royce automobile

For which of the following products would exclusive distribution be most appropriate? a) Gasoline b) Rolls Royce automobile c) Washing machine d) Laundry detergent e) Moderately priced luggage

b) Rolls Royce automobile

146. What function of physical distribution enables companies to compensate for dissimilar production and consumption rates and stabilize prices and availability of seasonal items? a) Materials handling b) Warehousing c) Transportation d) Inventory management e) Order processing

b) Warehousing

What function of physical distribution enables companies to compensate for dissimilar production and consumption rates and stabilize prices and availability of seasonal items? a) Materials handling b) Warehousing c) Transportation d) Inventory management e) Order processing

b) Warehousing

159. The Diamond Salt Company needs to transport 20 tons of salt from Baton Rouge, Louisiana, to Cincinnati. Which of the following methods would be the cheapest for transporting this cargo? a) Pipelines b) Waterways c) Trucks d) Airways e) Railroads

b) Waterways

63. A channel that includes both a manufacturers' agent and an industrial distributor is appropriate under which of the following circumstances? a) When the firm wants specialized personnel to follow up the work of the sales force b) When the marketer wishes to enter a new geographic market but does not wish to expand the existing sales force c) When only one or two channels of distribution are available for products d) When the sales force is large and the marketer is thinking of cutting it down e) When customers are highly concentrated in one geographic area

b) When the marketer wishes to enter a new geographic market but does not wish to expand the existing sales force

A channel that includes both a manufacturers' agent and an industrial distributor is appropriate under which of the following circumstances? a) When the firm wants specialized personnel to follow up the work of the sales force b) When the marketer wishes to enter a new geographic market but does not wish to expand the existing sales force c) When only one or two channels of distribution are available for products d) When the sales force is large and the marketer is thinking of cutting it down e) When customers are highly concentrated in one geographic area

b) When the marketer wishes to enter a new geographic market but does not wish to expand the existing sales force

88. Nike maintains a good deal of control over how its products are promoted, displayed, and sold. Because of this control, Nike would be appropriately described as the channel a) intermediary. b) captain. c) allocator. d) terminator. e) price leader.

b) captain.

Nike maintains a good deal of control over how its products are promoted, displayed, and sold. Because of this control, Nike would be appropriately described as the channel a) intermediary. b) captain. c) allocator. d) terminator. e) price leader.

b) captain.

90. When one company in a marketing channel has the ability to influence another member's goal achievement, the company has a) channel control. b) channel power. c) marketing leadership. d) a channel captain. e) distributive influence.

b) channel power.

When one company in a marketing channel has the ability to influence another member's goal achievement, the company has a) channel control. b) channel power. c) marketing leadership. d) a channel captain. e) distributive influence.

b) channel power.

143. Fansteel manufactures parts used in jet engines, which it sells to companies such as Boeing. When shipping products to Boeing, Fansteel seals the parts in 8' × 8' × 40' boxes for shipping to decrease loss and damage. This method of materials handling is a) unit loading. b) containerization. c) freight forwarding. d) unitized shipping. e) bonded packaging.

b) containerization.

Fansteel manufactures parts used in jet engines, which it sells to companies such as Boeing. When shipping products to Boeing, Fansteel seals the parts in 8' 8' 40' boxes for shipping to decrease loss and damage. This method of materials handling is a) unit loading. b) containerization. c) freight forwarding. d) unitized shipping. e) bonded packaging.

b) containerization.

30. A channel of distribution is defined as a group of individuals and organizations that a) consumes about one-half of every dollar spent on products in the United States. b) directs the flow of products from producers to customers. c) links producers to other marketing intermediaries. d) takes title to products and resells them. e) manages transportation and warehousing functions.

b) directs the flow of products from producers to customers.

A channel of distribution is defined as a group of individuals and organizations that a) consumes about one-half of every dollar spent on products in the United States. b) directs the flow of products from producers to customers. c) links producers to other marketing intermediaries. d) takes title to products and resells them. e) manages transportation and warehousing functions.

b) directs the flow of products from producers to customers.

120. At the annual managers' planning conference, Jackie Conrad asks the other managers to consider her proposal to increase the quantity of inventory to a five-week supply in order to increase the percentage of completely filled customer orders from 85 percent to 90 percent. She states that she is willing to trade off the ___________ incurred for the positive effect of better customer service. a) lower response time b) higher warehousing costs c) higher transportation costs d) higher order processing costs e) higher performance levels

b) higher warehousing costs

75. When Busch Light Beer was introduced as part of the Anheuser-Busch product line, the company most likely used ___________ distribution. a) horizontal b) intensive c) selective d) agent e) exclusive

b) intensive

When Busch Light Beer was introduced as part of the Anheuser-Busch product line, the company most likely used ___________ distribution. a) horizontal b) intensive c) selective d) agent e) exclusive

b) intensive

77. Candy bars and chewing gum are most likely to be distributed through _____ and _____. a) selective distribution; multiple channels b) intensive; dual distribution c) strategic channel alliances; intensive d) exclusive; a single channel e) dual distribution; convenience channels

b) intensive; dual distribution

Candy bars and chewing gum are most likely to be distributed through _____ and _____. a) selective distribution; multiple channels b) intensive; dual distribution c) strategic channel alliances; intensive d) exclusive; a single channel e) dual distribution; convenience channels

b) intensive; dual distribution

86. Because Coke is such a popular product and the company is so powerful, Coca-Cola is in a position to exert considerable control over channel structures and the way Coke is marketed. This is an illustration of channel ___________ in the distribution channel. a) conflict b) leadership c) dominance d) cooperation e) negotiation

b) leadership

Because Coke is such a popular product and the company is so powerful, Coca-Cola is in a position to exert considerable control over channel structures and the way Coke is marketed. This is an illustration of channel ___________ in the distribution channel. a) conflict b) leadership c) dominance d) cooperation e) negotiation

b) leadership

33. Most marketing channels have marketing intermediaries. A marketing intermediary's role is to a) link wholesalers to other wholesalers. b) link producers to other middlemen or to consumers. c) always sell products to wholesalers. d) not take title to products. e) always sell products to retailers.

b) link producers to other middlemen or to consumers.

Most marketing channels have marketing intermediaries. A marketing intermediary's role is to a) link wholesalers to other wholesalers. b) link producers to other middlemen or to consumers. c) always sell products to wholesalers. d) not take title to products. e) always sell products to retailers.

b) link producers to other middlemen or to consumers.

133. The result of an inventory stockout is usually an increase in a) inventory costs. b) lost sales. c) net profit. d) customers. e) prices.

b) lost sales.

The result of an inventory stockout is usually an increase in a) inventory costs. b) lost sales. c) net profit. d) customers. e) prices.

b) lost sales.

65. An independent businessperson who is paid a commission to sell complementary products of different producers in an assigned territory without actually taking title of the merchandise is a(n) a) sole intermediary. b) manufacturers' agent. c) producers' broker. d) industrial distributor. e) channel facilitator.

b) manufacturers' agent.

An independent businessperson who is paid a commission to sell complementary products of different producers in an assigned territory without actually taking title of the merchandise is a(n) a) sole intermediary. b) manufacturers' agent. c) producers' broker. d) industrial distributor. e) channel facilitator.

b) manufacturers' agent.

29. A group of organizations and individuals that directs the flow of products from producers to the ultimate consumers is called a a) free trade association b) marketing channel c) channel of suppliers d) production line-up e) chain of retailers

b) marketing channel

A group of organizations and individuals that directs the flow of products from producers to the ultimate consumers is called a a) free trade association b) marketing channel c) channel of suppliers d) production line-up e) chain of retailers

b) marketing channel

41. Eliminating a wholesaler from a marketing channel will a) cut costs and lower prices. b) not eliminate the functions performed by that wholesaler. c) eliminate the functions performed by that wholesaler. d) lead to lower costs but higher prices. e) reduce channel conflict.

b) not eliminate the functions performed by that wholesaler.

Eliminating a wholesaler from a marketing channel will a) cut costs and lower prices. b) not eliminate the functions performed by that wholesaler. c) eliminate the functions performed by that wholesaler. d) lead to lower costs but higher prices. e) reduce channel conflict.

b) not eliminate the functions performed by that wholesaler.

127. The order-processing task that involves verifying product availability, checking prices and customer credit ratings, and filling orders is a) warehousing. b) order handling. c) information processing. d) materials handling. e) order entry.

b) order handling.

The order-processing task that involves verifying product availability, checking prices and customer credit ratings, and filling orders is a) warehousing. b) order handling. c) information processing. d) materials handling. e) order entry.

b) order handling.

89. Few supermarkets would try to replace a national brand of baby food with their own brand. Assuming that this is true, we have a good example of channel leadership by a) wholesalers. b) producers. c) retailers. d) agents. e) brokers.

b) producers.

Few supermarkets would try to replace a national brand of baby food with their own brand. Assuming that this is true, we have a good example of channel leadership by a) wholesalers. b) producers. c) retailers. d) agents. e) brokers.

b) producers.

138. All of the following are concepts used in a just-in-time inventory management system except a) supplies arrive just as they are needed. b) safety stock is kept to reduce stockouts. c) much waste is eliminated. d) purchases are made in small quantities. e) purchases are made frequently.

b) safety stock is kept to reduce stockouts.

All of the following are concepts used in a just-in-time inventory management system except a) supplies arrive just as they are needed. b) safety stock is kept to reduce stockouts. c) much waste is eliminated. d) purchases are made in small quantities. e) purchases are made frequently.

b) safety stock is kept to reduce stockouts.

72. Fragile products that require special handling are more likely to be distributed through a) longer channels. b) shorter channels. c) Channel D. d) Channel H. e) exclusive outlets.

b) shorter channels.

Fragile products that require special handling are more likely to be distributed through a) longer channels. b) shorter channels. c) Channel D. d) Channel H. e) exclusive outlets.

b) shorter channels.

38. Possession utility is best described as a) products being available in places where the customers wish to purchase them. b) the customer having access to the product to use now or store and use later. c) having a company's products available when a customer needs them. d) being able to legally own a product despite restrictions on trade. e) getting the products to the consumers in as short of time as possible for ownership.

b) the customer having access to the product to use now or store and use later.

100. The reasons a vertically integrated channel can be more effective against competition is because of all of the following except a) the consolidation of power. b) tightly controlled and bureaucratic management style. c) the ability to inhibit competitors. d) the sharing of responsibilities and information. e) increased bargaining power.

b) tightly controlled and bureaucratic management style.

The reasons a vertically integrated channel can be more effective against competition is because of all of the following except a) the consolidation of power. b) tightly controlled and bureaucratic management style. c) the ability to inhibit competitors. d) the sharing of responsibilities and information. e) increased bargaining power.

b) tightly controlled and bureaucratic management style.

Having products available when the customer wants them is called a) place utility. b) time utility. c) availability. d) possession utility. e) chronavailability.

b) time utility.

102. When Benetton, the Italian sportswear designer and manufacturer, decided to open its own specialty shops to sell its merchandise, the firm was engaging in a) channel repetition. b) vertical channel integration. c) channel conflict. d) horizontal channel integration. e) channel expansion.

b) vertical channel integration.

When Benetton, the Italian sportswear designer and manufacturer, decided to open its own specialty shops to sell its merchandise, the firm was engaging in a) channel repetition. b) vertical channel integration. c) channel conflict. d) horizontal channel integration. e) channel expansion.

b) vertical channel integration.

101. Bennetton, a leading manufacturer of knitwear in Italy, expanded its operations to include retail outlets in the United States and South America. This type of integration is called a) horizontal. b) vertical. c) multilevel. d) retail. e) merchandising.

b) vertical.

Bennetton, a leading manufacturer of knitwear in Italy, expanded its operations to include retail outlets in the United States and South America. This type of integration is called a) horizontal. b) vertical. c) multilevel. d) retail. e) merchandising.

b) vertical.

A marketing channel is defined as a group of individuals and organizations that Select one: a. consumes about one-half of every dollar spent on products in the United States. b. directs the flow of products from producers to customers. c. links producers to other marketing intermediaries. d. manages transportation and warehousing functions. e. takes title to products and resells them.

b.

Netflix sells its movie services using its website while Red Box sells its movie services using vending machines. From a customer's point of view, Netflix is using _________________ and Red Box is using ____________ . Select one: a. an Internet wholesaler; a retailer. b. a direct-marketing channel; a type of retailer. c. an Internet wholesaler; a slightly longer channel. d. a direct-marketing channel; an agent. e. a direct marketing channel; a slightly shorter channel.

b.

Which of the following levels of market coverage is most appropriate for products that have a high replacement rate, require almost no service, and are bought based on price cues? a. selective distribution b. intensive distribution c. exclusive distribution d. digital distribution e. physical distribution

b. intensive distribution

Which of the following involves planning, implementing, and controlling the efficient and effective flow and storage of products and information from the point of origin to consumption to meet customers' needs and wants? a. physical distribution b. logistics management c. supply-chain management d. supply management e. operations management

b. logistics management

The use of a variety of marketing channels to ensure maximum distribution is best called _____________. a. supply chain alliances b. multichannel distribution c. digital distribution d. logistics e. channel integration

b. multichannel distribution

Hulu enables consumers to watch select TV shows whenever they want instead of when they are broadcast. This illustrates _______________. a. possession utility b. time utility c. form utility d. technological utility e. place utility

b. time utility

Which of the following provides time utility by enabling firms to compensate for dissimilar production and consumption rates? a. inventory management b. warehousing c. materials handling d. transportation e. order processing

b. warehousing

When Gabriella logs on to Dell's website, she sees a notebook model priced will below $1,000. As she continues through the site to view the other options, she realizes the first one she saw was the cheapest model available, but she of course wants more features. Dell is utilizing

bait pricing

The oldest form of exchange trading of products is known as

barter

Karen has been a buyer for Ocean Mist, a large cranberry processor, for several years. Believing that she knows a great deal about cranberries, their growers, and processors, she decides to go into business by herself. Karen wants to assume no risks of spoilage or price fluctuations. She sees herself as simply bringing the growers and processors together. Her new company would be a _____.

broker

When Mia and Shane are planning their honeymoon, their travel agent tells them that if they buy a special package, their trip to Paris includes meals, tickets to the theater, and a rental car in addition to airfare and a hotel. This is an example of the use of

bundle pricing

Some grocery stores collect data on competitive prices

by using full-time comparison shoppers

Some grocery stores collect data on competitive prices

by using full-time comparison shoppers.

A retailer is an organization that purchases products for the purpose of reselling them to a. other retail organizations. b. the government. c. ultimate consumers. d. wholesalers. e. nonprofit organizations.

c

A small self-service store that is open long hours and carries a narrow product assortment in convenient locations is best described as a a. discount store. b. department store. c. convenience store. d. supermarket. e. category killer.

c

A(n) ___________ is a very large specialty store that competes on the basis of lower prices and enormous product availability. a. warehouse club b. off-price retailer c. category killer d. traditional specialty store e. super center

c

Any organization that purchases products for the purpose of reselling them to ultimate consumers is a(n) a. wholesaler. b. shopping center. c. retailer. d. intermediary. e. producer.

c

Retail facilities located in big, low-cost buildings with large on-premise inventories and minimal services are called a. catalog showrooms. b. category killers. c. warehouse showrooms. d. warehouse clubs. e. display outlets.

c

168. In which of the following situations is dual distribution likely to be determined illegal? a) A manufacturer sells its product online and through independent and company-owned retail outlets. b) A manufacturer opens its own retail outlets across the country and sells its entire line of products. c) A producer uses company outlets to dominate independent retailers that carry its products. d) A producer sells its products both to wholesalers who deal with small and medium-sized retailers and directly to retailers. e) A producer charges higher prices at its company-owned retail outlets than independent retailers charge for the same products.

c) A producer uses company outlets to dominate independent retailers that carry its products.

In which of the following situations is dual distribution likely to be determined illegal? a) A manufacturer sells its product online and through independent and company-owned retail outlets. b) A manufacturer opens its own retail outlets across the country and sells its entire line of products. c) A producer uses company outlets to dominate independent retailers that carry its products. d) A producer sells its products both to wholesalers who deal with small and medium-sized retailers and directly to retailers. e) A producer charges higher prices at its company-owned retail outlets than independent retailers charge for the same products.

c) A producer uses company outlets to dominate independent retailers that carry its products.

50. The marketing channel of producer to retailer to consumer is most likely to be used by producers of which of the following products? a) Chewing gum b) Tobacco c) Automobiles d) Hardware e) Saltine crackers

c) Automobiles

The marketing channel of producer to retailer to consumer is most likely to be used by producers of which of the following products? a) Chewing gum b) Tobacco c) Automobiles d) Hardware e) Saltine crackers

c) Automobiles

26. Where do supply chains start? a) Raw materials b) Suppliers c) Customers d) Producers e) Retailers

c) Customers

132. Greg Braddock is a physical distribution manager. He is currently developing and maintaining assortments of products that are adequate for customer demand. In which stage in the physical distribution system is Greg currently involved? a) Order processing b) Materials handling c) Inventory management d) Transportation e) Warehousing

c) Inventory management

Greg Braddock is a physical distribution manager. He is currently developing and maintaining assortments of products that are adequate for customer demand. In which stage in the physical distribution system is Greg currently involved? a) Order processing b) Materials handling c) Inventory management d) Transportation e) Warehousing

c) Inventory management

71. Which of the following is least likely to be a factor affecting the selection of marketing channels? a) Customer characteristics b) Product attributes c) Product packaging d) Competition e) Environmental forces

c) Product packaging

Which of the following is least likely to be a factor affecting the selection of marketing channels? a) Customer characteristics b) Product attributes c) Product packaging d) Competition e) Environmental forces

c) Product packaging

59. Which of the following is most likely to be a product stocked by an industrial distributor? a) Tires b) Exercise equipment c) Screws d) Countertops e) Bananas

c) Screws

Which of the following is most likely to be a product stocked by an industrial distributor? a) Tires b) Exercise equipment c) Screws d) Countertops e) Bananas

c) Screws

134. Anna Wren, sales manager for Pacific Lumber, tells Jason Peoples, the firm's inventory manager, that the firm's failure to have adequate supplies of pressure-treated lumber on hand has cost the firm $175,000 in lost sales. This figure represents which of the following inventory management costs? a) Carrying b) Replenishment c) Stockout d) Safety stock e) Reorder

c) Stockout

Anna Wren, sales manager for Pacific Lumber, tells Jason Peoples, the firm's inventory manager, that the firm's failure to have adequate supplies of pressure-treated lumber on hand has cost the firm $175,000 in lost sales. This figure represents which of the following inventory management costs? a) Carrying b) Replenishment c) Stockout d) Safety stock e) Reorder

c) Stockout

174. If an intermediary wants to carry Tide detergent and Procter & Gamble agrees only if the supplier purchases P&G's entire line of detergents as well, the company is engaging in which of the following channel management practices? a) Exclusive dealing b) Dual distribution c) Tying agreement d) Refusal to deal e) Restricted sales territories

c) Tying agreement

If an intermediary wants to carry Tide detergent and Procter & Gamble agrees only if the supplier purchases P&G's entire line of detergents as well, the company is engaging in which of the following channel management practices? a) Exclusive dealing b) Dual distribution c) Tying agreement d) Refusal to deal e) Restricted sales territories

c) Tying agreement

44. Customers who purchase computer laptops from manufacturer websites are acquiring products through a) the most efficient channel of distribution. b) the most common type of marketing channel. c) a direct-marketing channel. d) a business-to-business channel of distribution. e) an indirect-marketing channel.

c) a direct-marketing channel.

64. Rob Stevens is the head of a company that produces computer software for production scheduling. The firm is small and presently does not generate enough volume to justify hiring a sales force. The firm is probably using ___________ to maintain contact with the firms using its products. a) wholesalers b) brokers c) agents d) merchants e) retailers

c) agents

Rob Stevens is the head of a company that produces computer software for production scheduling. The firm is small and presently does not generate enough volume to justify hiring a sales force. The firm is probably using ___________ to maintain contact with the firms using its products. a) wholesalers b) brokers c) agents d) merchants e) retailers

c) agents

62. A producer is not likely to receive _____ from an industrial distributor. a) selling activities in local markets b) market information about consumers c) aggressive promotion of its brand d) reduced capital requirements e) a reduced financial burden from customers

c) aggressive promotion of its brand

A producer is not likely to receive _____ from an industrial distributor. a) selling activities in local markets b) market information about consumers c) aggressive promotion of its brand d) reduced capital requirements e) a reduced financial burden from customers

c) aggressive promotion of its brand

158. Kolder Inc. is using a new just-in-time inventory management system that requires ordering smaller quantities of parts more frequently. The success of its business depends significantly on receiving these parts very quickly, so like many other companies using just-in-time, Kolder Inc. is relying more on the ______ mode of transportation despite its expense. a) railroad b) pipeline c) airway d) truck e) waterway

c) airway

Kolder Inc. is using a new just-in-time inventory management system that requires ordering smaller quantities of parts more frequently. The success of its business depends significantly on receiving these parts very quickly, so like many other companies using just-in-time, Kolder Inc. is relying more on the ______ mode of transportation despite its expense. a) railroad b) pipeline c) airway d) truck e) waterway

c) airway

If a wholesaler continually emphasizes and promotes one company's products over a competing company's products, _____ is likely to result. a) channel cooperation b) vertical channel integration c) channel conflict d) horizontal channel integration e) channel leadership.

c) channel conflict

The main objective of physical distribution should be to a) decrease costs while increasing market coverage. b) increase service and market coverage. c) decrease costs while increasing service. d) increase market coverage and channel power. e) balance costs and market coverage.

c) decrease costs while increasing service.

129. A commonly used computerized means of integrating order processing with production, inventory, accounting, and transportation is a) web-based inventory management. b) just-in-time. c) electronic data interchange. d) universal product codes. e) activity based management.

c) electronic data interchange.

A commonly used computerized means of integrating order processing with production, inventory, accounting, and transportation is a) web-based inventory management. b) just-in-time. c) electronic data interchange. d) universal product codes. e) activity based management.

c) electronic data interchange.

91. If Ralston Purina forced Kroger's grocery chain to place all of its products in the stores' most favorable locations, it would be a) demonstrating sound channel leadership. b) insisting on exclusive exposure. c) exercising channel power. d) minimizing channel conflict. e) creating a coordinate system.

c) exercising channel power.

If Ralston Purina forced Kroger's grocery chain to place all of its products in the stores' most favorable locations, it would be a) demonstrating sound channel leadership. b) insisting on exclusive exposure. c) exercising channel power. d) minimizing channel conflict. e) creating a coordinate system.

c) exercising channel power.

110. To expand the number of its retail outlets in the Washington, D.C., area, the Dress Barn bought out a small chain of women's apparel stores in northern Virginia. This type of integration is called a) vertical. b) retail. c) horizontal. d) backward. e) forward.

c) horizontal.

To expand the number of its retail outlets in the Washington, D.C., area, the Dress Barn bought out a small chain of women's apparel stores in northern Virginia. This type of integration is called a) vertical. b) retail. c) horizontal. d) backward. e) forward.

c) horizontal.

58. Jeff Wood's company buys machine tools from large producers and sells them to several Midwestern manufacturing companies. The company Jeff works for carries inventories of the tools, which reduces capital requirements for the producers. Jeff's company is an example of a(n) ___________ in a distribution channel. a) direct distributor b) manufacturers' agent c) industrial distributor d) producers' agent e) wholesalers' agent

c) industrial distributor

76. Producers of convenience products such as soft drinks, toothpaste, and breath mints are most likely to use _____ distribution. a) selective b) extensive c) intensive d) exclusive e) demand-based

c) intensive

78. Sales are most likely to have a direct relationship to product availability for products that use _____ distribution. a) selective b) dual c) intensive d) extensive e) exclusive

c) intensive

Sales are most likely to have a direct relationship to product availability for products that use _____ distribution. a) selective b) dual c) intensive d) extensive e) exclusive

c) intensive

131. Because of the significant investment many companies have in the products they sell to customers, they must develop and maintain adequate varieties of products to meet their customers' needs. This is called a) just-in-time. b) order processing. c) inventory management. d) merchandise stocking. e) logistical management.

c) inventory management.

144. Managers and consultants at Colgate-Palmolive are meeting to discuss better ways to package its products to reduce damage and make loading more efficient. The primary function they are trying to improve is a) warehousing. b) inventory management. c) materials handling. d) cycle time. e) containerization.

c) materials handling.

Managers and consultants at Colgate-Palmolive are meeting to discuss better ways to package its products to reduce damage and make loading more efficient. The primary function they are trying to improve is a) warehousing. b) inventory management. c) materials handling. d) cycle time. e) containerization.

c) materials handling.

Order processing, inventory management, materials handling, warehousing, and transportation are the activities that define a) wholesaling. b) retailing. c) physical distribution. d) channel management. e) drop shipping.

c) physical distribution.

Without wholesalers and other intermediaries, a) most products would be much less expensive because fewer companies would be handling the product. b) products would be cheaper because the functions of intermediaries would be eliminated. c) products would likely be more expensive due to the use of less efficient channel members. d) products would never be able to make it to the ultimate consumer at any price without passing through intermediaries. e) many products would be more expensive because retailers would expect more profit.

c) products would likely be more expensive due to the use of less efficient channel members.

171. Tying agreements occur when a a) producer distributes the same product through two or more different channels. b) manufacturer forbids an intermediary to carry products of competing manufacturers. c) supplier furnishes a product to a channel member with the stipulation that the channel member must purchase other products as well. d) producer refuses to deal to channel members that seem unethical or illegitimate. e) manufacturer prohibits intermediaries from selling its products outside designated sales territories.

c) supplier furnishes a product to a channel member with the stipulation that the channel member must purchase other products as well.

Tying agreements occur when a a) producer distributes the same product through two or more different channels. b) manufacturer forbids an intermediary to carry products of competing manufacturers. c) supplier furnishes a product to a channel member with the stipulation that the channel member must purchase other products as well. d) producer refuses to deal to channel members that seem unethical or illegitimate. e) manufacturer prohibits intermediaries from selling its products outside designated sales territories.

c) supplier furnishes a product to a channel member with the stipulation that the channel member must purchase other products as well.

112. The notion of physical distribution is a) speed of delivery. b) used only by manufacturers. c) the movement of products from producers to end users. d) primarily the wholesaler's responsibility. e) an assessment of distribution costs.

c) the movement of products from producers to end users.

The notion of physical distribution is a) speed of delivery. b) used only by manufacturers. c) the movement of products from producers to end users. d) primarily the wholesaler's responsibility. e) an assessment of distribution costs.

c) the movement of products from producers to end users.

When Busch Light Beer was introduced as part of the Anheuser-Busch product line, the company most likely used _____________ distribution. Select one: a. exclusive b. horizontal c. intensive d. selective e. agent

c.

Which of the following levels of market coverage is desirable when a special effort, such as customer service from a channel member, is important to customers? a. intensive distribution b. exclusive distribution c. selective distribution d. digital distribution e. multichannel distribution

c. selective distribution

Which of the following integrates the functions of operations management, logistics management, supply management, and marketing channel management so that products are produced and distributed in the right quantities, to the right locations, and at the right times? a. multichannel distribution b. 3rd party logistics c. supply-chain management d. physical distribution e. channel integration

c. supply chain management

Maintaining or increasing market share

can be achieved even if industry sales are flat or decreasing

A market share objective

can be used effectively whether total industry sales are rising or falling

Lexmark sells some of its color printers for about $100, but the refill cartridges cover over $30 each. Lexmark's pricing strategy would be best labeled as

captive pricing

Lexmark sells some of its color printers for about $100, but the refill cartridges cost over $30 each. Lexmark's pricing strategy would be best labeled as

captive pricing.

Pricing the basic product in a product line low while pricing related items at a higher level is called

captive pricing.

Gambrell Designs thinks its new product, the Automatic Dog Walker, will have a short product life cycle; therefore, its marketing department sets its primary pricing objective as

cash flow

Gambrell Designs thinks its new product, the Automatic Dog Walker, will have a short product life cycle; therefore, its marketing department sets its primary pricing objective as

cash flow.

Andrea owns and operates a small novelty store that sells home décor, gifts, jewelry, and gourmet snacks. Because her business is very small, many wholesalers do not deal with her, therefore she relies primarily on _____ for the survival of her business.

cash-and-carry wholesalers

Companies that focus on particular product categories and rely on everyday low pricing to acquire a large market share through aggressive and competitive pricing strategies are often referred to as

category killers

Question Companies that focus on particular product categories and rely on everyday low pricing to acquire a large market share through aggressive and competitive pricing strategies are often referred to as

category killers.

A retail strategy of managing groups of similar, often substitutable products produced by different manufacturers is called _____.

category management

Showing a product's price along with its previous price, the price of a competing brand, or the price at another retail outlet is called

comparison discounting

Showing a product's price along with its previous price, the price of a competing brand, or the price at another retail outlet is called

comparison discounting.

When products in an industry are relatively homogeneous and price is a key purchase consideration,

competition based-pricing becomes more important

If PepsiCo sets its twelve-pack price at $3.99 to match the price charged by Coca-Cola, Pepsi is using which of the following pricing methods?

competition-based

If General Mills looks at Kellogg's cereal prices as the primary method of determining its own prices, General Mills is using

competition-based pricing

If General Mills looks at Kellogg's cereal prices as the primary method of determining its own prices, General Mills is using

competition-based pricing.

Firestone allows companies like Sears and Discount Tire to distribute and discount its tires. This action significantly increases the possibility of channel ____ with independent Firestone dealers. a. conflict b. power c. leadership d. communication e. understanding

conflict

Goodyear allows companies like Sears and Discount Tire to distribute and discount its tires. This action significantly increases the possibility of channel _____ with independent Goodyear dealers.

conflict

Piggyback, fishyback, and birdyback are terms usually associated with gaining efficiency in shipping through:

containerization.

A _____ is a small, self-service store that is open long hours and carries a narrow assortment of products, usually items such as soft drinks and other beverages, snacks, news-papers, tobacco, and gasoline, as well as services such as ATMs.

convenience store

overall channel goals and individual channel member goals cannot be achieved together without: -conflict -captains -leadership -cooperation -tying agreements

cooperation

The Limited, which produces and retails clothing products in a coordinated channel, is an example of a(n):

corporate vertical marketing system

For custom-made equipment or commercial construction projects, which pricing method is most likely used?

cost-plus

Question When a seller's costs are usually determined during or after a product is made and then a specified percentage or dollar amount is added to the cost to establish a price, an organization is using _____ pricing.

cost-plus

Steinway produces concert grand pianos, often using the custom materials and designs desired by a specific customer. The average price of these pianos runs about $50,000 depending on the exact piano. What type of pricing does Steinway most likely use for these pianos?

cost-plus

The federal government often uses ___ pricing when it grants defense contracts

cost-plus

The federal government often uses ____ pricing when it grants defense contracts.

cost-plus

When a seller's costs are usually determined during or after a product is made and then specified percentage or dollar amount is added to the cost to establish a price, an organization is using ___ pricing.

cost-plus

Marketers generally view _____ as the minimum price a product can be sold for.

costs

All of the following are psychological techniques except

customary pricing

Goods that are priced primarily based on the way they have always been priced are examples of

customary pricing

Question Goods that are priced primarily based on the way they have always been priced are examples of

customary pricing.

The driving force behind marketing channel decisions should be:

customer satisfaction.

Location is important to a retailer because a. suppliers charge more to service stores in certain trading areas. b. a desirable location appeals to consumers' emotions. c. location is the major determinant of store image. d. location determines the trading area from which the store must draw its customers. e. convenient location is an essential customer service element.

d

Maintaining or increasing market share Answers: a. is directly tied to leading an industry in product quality. b. depends on the overall growth of the total industry. c. is a profit-related objective based on price. d. can be achieved even if industry sales are flat or decreasing. e. is an infrequently used pricing objective in most industries.

d

Many cities are revitalizing their downtown areas by bringing in new retail establishments. They are building new civic structures, passing ordinances that encourage the restoration of historic buildings, and developing mixed-use structures that include retail, residential, and other uses. These areas are best described as a. neighborhood shopping centers. b. community shopping centers. c. traditional shopping centers. d. traditional business districts. e. free-standing structures.

d

Matt's girlfriend tells him she wants a cashmere sweater for Christmas. Matt decides to go to a store that provides the best possible selection of sweaters. His best choice would be to shop at a________ store. a. department b. catalog c. discount d. traditional specialty e. convenience

d

Specialty stores offer ______ than department stores. a. greater varieties of product lines and higher prices b. lower prices and higher quality c. more product depth and less customer service d. more sales expertise and better selections e. more frequent price discounts and less selection

d

Under Armour is establishing a ______ pricing objective to maintain or increase its product's sales in relation to total industry sales. Answers: a. status quo b. survival c. return on investment d. market share e. product quality

d

What is the primary distinction between superstores and hypermarkets? a. Superstores offer a wider variety of products than hypermarkets. b. Superstores offer low prices while hypermarkets have moderate to high prices. c. Hypermarkets have fewer departments but deeper product lines than superstores. d. Hypermarkets are larger and have more types of products than superstores. e. These two types of retailers carry very different types of products.

d

151. Garcia Wholesale Plumbing has seen its sales in the Southeast triple in the past two years. Materials handling director Barb Peterson announces plans to the board for an Atlanta facility that will combine shipments received from Garcia's 25 suppliers for nearly immediate shipment to plumbing stores and contractors in the region. This new facility would be best classified as which of the following? a) Sales office b) Public warehouse c) Field public warehouse d) Distribution center e) Bonded warehouse

d) Distribution center

Garcia Wholesale Plumbing has seen its sales in the Southeast triple in the past two years. Materials handling director Barb Peterson announces plans to the board for an Atlanta facility that will combine shipments received from Garcia's 25 suppliers for nearly immediate shipment to plumbing stores and contractors in the region. This new facility would be best classified as which of the following? a) Sales office b) Public warehouse c) Field public warehouse d) Distribution center e) Bonded warehouse

d) Distribution center

Select the greatest advantage of horizontal channel integration. a) The flexibility of the channel is decreased. b) The markets are more heterogeneous. c) The expanded number of units is coordinated. d) Efficiencies in advertising, marketing research, and purchasing are increased. e) Planning and research are increased to cope with increased competition.

d) Efficiencies in advertising, marketing research, and purchasing are increased.

83. Honey Farms is a maker of fine chocolates. The company's latest product, Fudge-Dipped Strawberries, is the premier product in its Fudge-Dipped line. The product is very expensive and targeted to upscale consumers. Which form of distribution would Honey Farms be likely to use for its new product? a) Intensive b) Selective c) Targeted d) Exclusive e) Premier

d) Exclusive

Honey Farms is a maker of fine chocolates. The company's latest product, Fudge-Dipped Strawberries, is the premier product in its Fudge-Dipped line. The product is very expensive and targeted to upscale consumers. Which form of distribution would Honey Farms be likely to use for its new product? a) Intensive b) Selective c) Targeted d) Exclusive e) Premier

d) Exclusive

139. Justin Abercrombie attends a seminar on just-in-time inventory management. Excited about the idea, he returns to his office at Atlanta Auto Supply to start designing such an inventory system. Which of the following steps must Justin take as he institutes this change? a) Increase safety stock b) Raise the reorder point c) Reduce customer service standard d) Increase order frequency e) Reduce estimates of inventory turnover rate

d) Increase order frequency

Justin Abercrombie attends a seminar on just-in-time inventory management. Excited about the idea, he returns to his office at Atlanta Auto Supply to start designing such an inventory system. Which of the following steps must Justin take as he institutes this change? a) Increase safety stock b) Raise the reorder point c) Reduce customer service standard d) Increase order frequency e) Reduce estimates of inventory turnover rate

d) Increase order frequency

125. What are the three primary tasks of order processing? a) Order receipt, order delivery, order follow-up b) Order handling, inventory management, order delivery c) Materials handling, warehousing, order delivery d) Order handling, order entry, order delivery e) Order receipt, order checking, order delivery

d) Order handling, order entry, order delivery

53. Which of the following is the most commonly used channel for distributing business products? a) Producer, agents, industrial distributors, organizational buyers b) Producer, industrial distributors, organizational buyers c) Producer, agents, organizational buyers d) Producer, organizational buyers e) Industrial distributors, organizational buyers

d) Producer, organizational buyers

Which of the following is the most commonly used channel for distributing business products? a) Producer, agents, industrial distributors, organizational buyers b) Producer, industrial distributors, organizational buyers c) Producer, agents, organizational buyers d) Producer, organizational buyers e) Industrial distributors, organizational buyers

d) Producer, organizational buyers

49. Large retailers such as J.C. Penney's and Target are most likely to participate in which of the following channels? a) Producer, industrial distributors, retailers, consumers b) Producer, consumers c) Producer, wholesalers, retailers, consumers d) Producer, retailers, consumers e) Producer, agents, wholesalers, retailers, consumers

d) Producer, retailers, consumers

Large retailers such as J.C. Penney's and Target are most likely to participate in which of the following channels? a) Producer, industrial distributors, retailers, consumers b) Producer, consumers c) Producer, wholesalers, retailers, consumers d) Producer, retailers, consumers e) Producer, agents, wholesalers, retailers, consumers

d) Producer, retailers, consumers

51. Nationally distributed consumer convenience products are most likely distributed through which of the following channels? a) Producer, consumers b) Producer, agents, wholesalers, retailers, consumers c) Producer, wholesalers, consumers d) Producer, wholesalers, retailers, consumers e) Producer, industrial distributor, wholesalers, retailers, consumers

d) Producer, wholesalers, retailers, consumers

152. What is the most expensive physical distribution function? a) Warehousing b) Order processing c) Inventory management d) Transportation e) Materials handling

d) Transportation

What is the most expensive physical distribution function? a) Warehousing b) Order processing c) Inventory management d) Transportation e) Materials handling

d) Transportation

145. Which of the following physical distribution functions involves design and operation of facilities for storing goods? a) Order processing b) Materials handling c) Transportation d) Warehousing e) Inventory management

d) Warehousing

25. The supply chain includes a) producers, wholesalers, and retailers. b) suppliers, producers, intermediaries, and customers. c) suppliers and suppliers' suppliers. d) all entities that facilitate product distribution. e) buyers, seller, marketing intermediaries, and agents.

d) all entities that facilitate product distribution.

The supply chain includes a) producers, wholesalers, and retailers. b) suppliers, producers, intermediaries, and customers. c) suppliers and suppliers' suppliers. d) all entities that facilitate product distribution. e) buyers, seller, marketing intermediaries, and agents.

d) all entities that facilitate product distribution.

87. A single leader who controls and organizes a marketing channel is called a a) channel champion. b) distribution leader. c) marketing maverick. d) channel captain. e) lead distributor.

d) channel captain.

A single leader who controls and organizes a marketing channel is called a a) channel champion. b) distribution leader. c) marketing maverick. d) channel captain. e) lead distributor.

d) channel captain.

162. Piggyback, fishyback, and birdyback are terms usually associated with gaining efficiency in shipping through a) transit time. b) warehousing. c) packaging. d) containerization. e) lot sizes.

d) containerization.

Piggyback, fishyback, and birdyback are terms usually associated with gaining efficiency in shipping through a) transit time. b) warehousing. c) packaging. d) containerization. e) lot sizes.

d) containerization.

108. The Wicks 'N Sticks candle and gift marketing organization has a(n) ___________ vertical marketing system arrangement with its franchised retail store operations. a) corporate b) administered c) negotiated d) contractual e) horizontal

d) contractual

The Wicks 'N Sticks candle and gift marketing organization has a(n) ___________ vertical marketing system arrangement with its franchised retail store operations. a) corporate b) administered c) negotiated d) contractual e) horizontal

d) contractual

92. Overall channel goals and individual channel member goals cannot be achieved together without a) conflict. b) captains. c) leadership. d) cooperation. e) tying agreements.

d) cooperation.

Overall channel goals and individual channel member goals cannot be achieved together without a) conflict. b) captains. c) leadership. d) cooperation. e) tying agreements.

d) cooperation.

31. The driving force behind marketing channel decisions should be a) convenience. b) cost reduction. c) environmental concerns. d) customer satisfaction. e) quality.

d) customer satisfaction.

The driving force behind marketing channel decisions should be a) convenience. b) cost reduction. c) environmental concerns. d) customer satisfaction. e) quality.

d) customer satisfaction.

45. Select Comfort, a producer of adjustable air mattresses, sells most of its products through direct mail sales and Internet sales. This channel would be classified as a) direct distribution. b) producer, retailer, consumer. c) telemarketing. d) direct-marketing. e) indirect marketing.

d) direct-marketing.

Select Comfort, a producer of adjustable air mattresses, sells most of its products through direct mail sales and Internet sales. This channel would be classified as a) direct distribution. b) producer, retailer, consumer. c) telemarketing. d) direct-marketing. e) indirect marketing.

d) direct-marketing.

169. Ralph Lauren sells Polo products through department stores as well as in its own specialty shops. The designer uses ___________ as a channel strategy. a) channel extension b) intermediary exclusion c) broker utilization d) dual distribution e) channel diversification

d) dual distribution

Ralph Lauren sells Polo products through department stores as well as in its own specialty shops. The designer uses ___________ as a channel strategy. a) channel extension b) intermediary exclusion c) broker utilization d) dual distribution e) channel diversification

d) dual distribution

68. Many companies use more than one marketing channel to distribute their products to the same target market, a tactic called a) multiple channeling. b) strategic channel alliance. c) intensive distribution. d) dual distribution. e) market splitting.

d) dual distribution.

Many companies use more than one marketing channel to distribute their products to the same target market, a tactic called a) multiple channeling. b) strategic channel alliance. c) intensive distribution. d) dual distribution. e) market splitting.

d) dual distribution.

42. When three buyers purchase the products of three producers, nine transactions are required. If one intermediary serves both producers and buyers, the number of possible transactions is a) fifteen. b) five. c) eighteen. d) six. e) twenty.

d) six.

When three buyers purchase the products of three producers, nine transactions are required. If one intermediary serves both producers and buyers, the number of possible transactions is a) fifteen. b) five. c) eighteen. d) six. e) twenty

d) six.

106. In an administered VMS, informal coordination brings about a high level of interorganizational management. Nonetheless, a) decision making does not take into account the goals of the system. b) the channel members do not remain autonomous. c) the decision making is not coordinated. d) the channel members remain autonomous. e) the goals of the individual firms are not congruent with the goals of the system.

d) the channel members remain autonomous.

In an administered VMS, informal coordination brings about a high level of interorganizational management. Nonetheless, a) decision making does not take into account the goals of the system. b) the channel members do not remain autonomous. c) the decision making is not coordinated. d) the channel members remain autonomous. e) the goals of the individual firms are not congruent with the goals of the system.

d) the channel members remain autonomous.

142. Federal Mogul places several boxes of products or materials on pallets in order to load them efficiently using forklifts. This system of materials handling is called a) warehousing. b) containerization. c) group storage. d) unit loading. e) mechanized handling.

d) unit loading.

Federal Mogul places several boxes of products or materials on pallets in order to load them efficiently using forklifts. This system of materials handling is called a) warehousing. b) containerization. c) group storage. d) unit loading. e) mechanized handling.

d) unit loading.

A single leader who controls and organizes a marketing channel is called a Select one: a. channel champion. b. lead distributor. c. distribution leader. d. channel captain. e. marketing maverick.

d.

An independent businessperson who is paid a commission to sell complementary products of different producers in an assigned territory without actually taking title of the merchandise is a(n) Select one: a. producers' broker b. channel facilitator. c. sole intermidiary. d. manufacturers' agent. e. industrial distributor.

d.

Consumers receive the benefits of place utility when Select one: a. they have to travel excessively to obtain products they want. b. retailers remain open 24 hours a day. c. they make purchases with credit and debit cards. d. products are available in locations where consumers want to buy them. e. they can stock up on products they need but not use them right away.

d.

If Purina forced Kroger's grocery chain to place all of its products in the stores' most favorable locations, it would be Select one: a. minimizing channel conflict. b. insisting on exclusive exposure. c. creating a coordinate system. d. exercising channel power. e. demonstrating sound channel leadership.

d.

Starbucks has an agreement with Pepsi Co. through which Pepsi distributes Starbucks' coffee drink, Frappucino, to grocery stores and other retail outlets. This is an example of Select one: a. horizontal channel integration. b. channel leadership. c. dual distribution. d. a strategic channel alliance. e. exclusive distribution.

d.

Steelcase, Inc. markets furniture directly to businesses. This is an example of a(n) _____________ channel. Select one: a. equipment b. producer-to-industrial distributor-to-business buyer c. consumer d. Producer-to-business buyer e. producer-to-agent-to-business buyer

d.

The result of an inventory stockout usually results in ______________ . Select one: a. a decrease in prices. b. an increase in loyal customers. c. an increase in inventory costs. d. an increase in lost sales. e. an increase in net profit.

d.

In any one large metropolitan area, there is usually just one Ferrari luxury sports car dealer. This best illustrates ______________. a. multichannel distribution b. digital distribution c. selective distribution d. exclusive distribution e. intensive distribution

d. exclusive distribution

If PetSmart began to acquire very successful local pet stores, this would be an example of ______________. a. channel power b. digital integration c. vertical integration d. horizontal integration e. multichannel integration

d. horizontal integration

When you buy a new backpack from LLBean.com, which of the following consumer marketing channels are you using? a. producer to wholesaler to retailer to consumer b. producer to industrial distribution to consumer c. producer to retailer to consumer d. producer to consumer e. producer to agent to wholesaler to retailer to consumer

d. producer to consumer

Supply-chain management should begin with a focus on ____________. a. the government regulator b. the seller c. the producer d. the customer e. the marketing channel

d. the customer

Which of the following have special expertise in core physical distribution activities such as warehousing, transportation, inventory management, and information technology and can often perform these activities more efficiently? a. corporate VMS b. freight forwarders c. megacarriers d. third-party logistics firms e. channel captains

d. third-party logistics firms

Both the Federal Trade Commission Act and the Wheeler-Lea Act prohibit

deceptive pricing

A graph of the quantity of products marketers expect to sell at various prices if other factors remain constant is a

demand curve

If General Motors determines that it wants to sell 200,000 Chevrolet Acadias and sets the price at $29,500 because it knows that at that price it will reach that goal, the firm would be using a ___ pricing method

demand-based

If a product is priced based on how many or how few people want it at a particular time and place, ______ pricing is being used.

demand-based

If a product is priced based on how many or how few people want it at a particular time and place, ___ pricing is being used

demand-based

Question Amtrak prices its tickets so that it is less expensive to travel on weekends than during the week when there is heavy business travel. This illustrates ___________ pricing.

demand-based

During July and August, Lakewood Links Golf Course, located in South Carolina, offers weekday rates of $13 for a round of golf with a cart. During the rest of the year, the weekday rates are between $25 and $35. This is an example of the use of

demand-based PRICING

Suppose that Rayban is considering a new line of sunglasses that would be sold in major department stores. The new line would be positioned as a more distinctive brand than the typical glasses sold through department stores, and would be priced higher than other brands in the store, but a lower price line than the current Rayban lines that are sold through more selective stores. In determining the price for this sunglass line, Rayban wants to gather information about all brands sold in department stores and about customers' perceptions of those brands. Reference: Ref 20-1 Rayban has decided to promote the new sunglass line as an "affordable luxury" and plans significant promotional expenditures. With these objectives, which of the following should Rayban use to price its product line?

demand-based pricing

If a store has areas for men's apparel, women's apparel, housewares, cosmetics, and jewelry and competes mostly on the basis of service, it is most likely a(n) _____.

department store

Coca-Cola offers a line of soft drinks that includes Coca-Cola, Diet Coke, Coca-Cola Zero, Sprite, and Fanta. These drinks best illustrate Coca-Cola's product mix a. life cycle. b. width. c. length. d. depth. e. volume.

depth

When establishing prices, a marketer's first step is to

develop pricing objectives.

Executives in Japan decided to price Lexus luxury cars in the US at $55,000 while pricing them at $66,000 in their own country. This is an example of

differential

Many firms decide to charge various prices to individual customers for the same quality and quantity of product, a pricing strategy known as

differential pricing.

When a retailer advertises a product and makes it available for purchase through telephone or mail orders, the retailer is using _____.

direct-response marketing

A channel of distribution is defined as a group of individuals and organizations that:

directs the flow of products from producers to customers.

Sony management decided to use skimming as a pricing strategy for its newest line of HDTV sets. It should be aware that this strategy does NOT

discourage competitors from entering the market

Sony management decided to use skimming as a pricing strategy for its newest line of highdefinition television (HDTV) sets. It should be aware that this strategy does not

discourage competitors from entering the market.

For most firms in the United States, demand curves are

downward sloping

Apple makes its computers available through its own stores, its website, and some major retailers. This is an example of:

dual distribution

Del Monte markets ketchup for household use to supermarkets through grocery wholesalers. It markets ketchup for institutional use through industrial distributors and food brokers. Del Monte is using:

dual distribution.

Many companies use more than one marketing channel to distribute their products to the same target market, a tactic known as:

dual distribution.

A general merchandise retailer offers a. a single product line that is stocked in depth. b. few product lines with deep assortments of these lines. c. few choices of a large variety of product lines. d. products through the Internet and catalogs only. e. a variety of product lines that are stocked in depth.

e

Greg chooses to have his hunting equipment stored in a location away from the main part of town so that he can have a large parking lot and an outdoor range for customers to test out his merchandise. Greg's store is a. in a traditional business district. b. a rural shopping center. c. in a neighborhood shopping center. d. a non-traditional shopping center. e. a free-standing structure.

e

Which of the following is an example of a category killer? a. Sears b. Kmart c. Marshalls d. Lady Foot Locker e. Toys "R" Us

e

Which type of retailer generally accepts lower margins than traditional retailers in exchange for higher sales volume? a. Department stores b. Traditional specialty retailers c. Warehouse showrooms d. Direct marketers e. Discount stores

e

82. Product and target market characteristics usually determine the type of coverage a product receives. For which of the following products is selective distribution most appropriate? a) Gasoline b) Jaguar automobiles c) Cigarettes d) Laundry detergent e) Panasonic stereos

e) Panasonic stereos

Product and target market characteristics usually determine the type of coverage a product receives. For which of the following products is selective distribution most appropriate? a) Gasoline b) Jaguar automobiles c) Cigarettes d) Laundry detergent e) Panasonic stereos

e) Panasonic stereos

109. Select the greatest advantage of horizontal channel integration. a) The flexibility of the channel is decreased. b) The markets are more heterogeneous. c) The expanded number of units is coordinated. d) Efficiencies in advertising, marketing research, and purchasing are increased. e) Planning and research are increased to cope with increased competition.

e) Planning and research are increased to cope with increased competition.

140. What is the primary determinant in deciding how materials will be handled? a) Unit loading b) Containerization c) Customer preference d) Cost reduction e) Product characteristics

e) Product characteristics

What is the primary determinant in deciding how materials will be handled? a) Unit loading b) Containerization c) Customer preference d) Cost reduction e) Product characteristics

e) Product characteristics

61. Which of the following is an advantage of using an industrial distributor? a) These firms are easy to control because they work directly for the producers. b) Inventory holding costs are minimized because they can store inventory very cheaply. c) They are closer geographically to all of the producers' customers. d) They possess a high level of technical knowledge about their products. e) They help reduce a producer's financial burdens by extending credit to customers.

e) They help reduce a producer's financial burdens by extending credit to customers.

Which of the following is an advantage of using an industrial distributor? a) These firms are easy to control because they work directly for the producers. b) Inventory holding costs are minimized because they can store inventory very cheaply. c) They are closer geographically to all of the producers' customers. d) They possess a high level of technical knowledge about their products. e) They help reduce a producer's financial burdens by extending credit to customers.

e) They help reduce a producer's financial burdens by extending credit to customers.

153. ___________ adds time and place utility to a product by moving it from where it is made to where it is purchased and used. a) Warehousing b) Containerization c) Distribution d) Materials handling e) Transportation

e) Transportation

___________ adds time and place utility to a product by moving it from where it is made to where it is purchased and used. a) Warehousing b) Containerization c) Distribution d) Materials handling e) Transportation

e) Transportation

104. When channel members are linked by legal agreements that specify each member's rights and responsibilities, _______ exists. a) horizontal channel integration b) an administered VMS c) a corporate VMS d) a channel captain e) a contractual VMS

e) a contractual VMS

95. Goodyear allows companies like Sears and Discount Tire to distribute and discount its tires. This action significantly increases the possibility of channel ___________ with independent Goodyear dealers. a) understanding b) power c) leadership d) communication e) conflict

e) conflict

Goodyear allows companies like Sears and Discount Tire to distribute and discount its tires. This action significantly increases the possibility of channel ___________ with independent Goodyear dealers. a) understanding b) power c) leadership d) communication e) conflict

e) conflict

24. The Home Depot is trying to reduce transportation, information management, and administrative costs. To accomplish this goal, channel members need to a) work independently. b) avoid cooperation due to antitrust considerations. c) work with competitors and share information. d) increase competition among channel members. e) cooperate and accommodate one another's needs.

e) cooperate and accommodate one another's needs.

The Home Depot is trying to reduce transportation, information management, and administrative costs. To accomplish this goal, channel members need to a) work independently. b) avoid cooperation due to antitrust considerations. c) work with competitors and share information. d) increase competition among channel members. e) cooperate and accommodate one another's needs.

e) cooperate and accommodate one another's needs.

103. The Limited, which produces and retails clothing products in a coordinated channel, is an example of a(n) a) administered vertical marketing system. b) conventional marketing channel. c) channel network. d) contractual marketing channel. e) corporate vertical marketing system.

e) corporate vertical marketing system.

The Limited, which produces and retails clothing products in a coordinated channel, is an example of a(n) a) administered vertical marketing system. b) conventional marketing channel. c) channel network. d) contractual marketing channel. e) corporate vertical marketing system.

e) corporate vertical marketing system.

128. Order handling involves all of the following activities except that the a) credit department approves the purchase. b) order is transmitted to the warehouse. c) availability of product is verified. d) warehouse is instructed to fill the order. e) customer places a purchase order.

e) customer places a purchase order.

Order handling involves all of the following activities except that the a) credit department approves the purchase. b) order is transmitted to the warehouse. c) availability of product is verified. d) warehouse is instructed to fill the order. e) customer places a purchase order.

e) customer places a purchase order.

27. All members of the supply chain should determine their position in the chain, identify their partners and their roles, and establish partnerships whose focus is a) shifting costs to suppliers. b) maximizing costs. c) maximizing technology implementation. d) cooperation with competitors. e) customer relationships.

e) customer relationships.

All members of the supply chain should determine their position in the chain, identify their partners and their roles, and establish partnerships whose focus is a) shifting costs to suppliers. b) maximizing costs. c) maximizing technology implementation. d) cooperation with competitors. e) customer relationships.

e) customer relationships.

123. A goal of physical distribution is to reduce ______ or how long it takes to complete a process. a) time standards b) processing time c) production d) throughput measure e) cycle time

e) cycle time

A goal of physical distribution is to reduce ______ or how long it takes to complete a process. a) time standards b) processing time c) production d) throughput measure e) cycle time

e) cycle time

121. An important goal of physical distribution is reducing the time it takes to complete a) inventory management. b) outsourcing evaluation. c) electronic data interchange. d) order processing. e) cycle time.

e) cycle time.

An important goal of physical distribution is reducing the time it takes to complete a) inventory management. b) outsourcing evaluation. c) electronic data interchange. d) order processing. e) cycle time.

e) cycle time.

46. Many producers selling on the Internet are using a) several intermediaries. b) complex marketing channels. c) Internet wholesalers, retailers, and agents. d) supply chain channels. e) direct-marketing channels.

e) direct-marketing channels.

Many producers selling on the Internet are using a) several intermediaries. b) complex marketing channels. c) Internet wholesalers, retailers, and agents. d) supply chain channels. e) direct-marketing channels.

e) direct-marketing channels.

167. Dual distribution is characterized as a) illegal under the Robinson-Patman Act. b) distribution channels that typically carry exclusive products, such as Rolex watches. c) marketing environments that are highly competitive. d) manufacturers that forbid an intermediary to carry products of competing producers. e) distribution of one manufacturer's product through two or more different channel structures.

e) distribution of one manufacturer's product through two or more different channel structures.

Dual distribution is characterized as a) illegal under the Robinson-Patman Act. b) distribution channels that typically carry exclusive products, such as Rolex watches. c) marketing environments that are highly competitive. d) manufacturers that forbid an intermediary to carry products of competing producers. e) distribution of one manufacturer's product through two or more different channel structures.

e) distribution of one manufacturer's product through two or more different channel structures.

172. An arrangement where a producer forbids an intermediary to carry products made by competing manufacturers is called a) exclusive distribution. b) a tying agreement. c) refusal to deal. d) contractual VMS. e) exclusive dealing.

e) exclusive dealing.

173. Dell Computers is a nationally recognized manufacturer of computers for the small business and home markets. If Dell were to order one of its wholesalers not to carry any computer products other than those made by Dell, this arrangement would be called a) a tying contract. b) refusal to deal. c) a restricted sales territory. d) a restricted channel. e) exclusive dealing.

e) exclusive dealing.

An arrangement where a producer forbids an intermediary to carry products made by competing manufacturers is called a) exclusive distribution. b) a tying agreement. c) refusal to deal. d) contractual VMS. e) exclusive dealing.

e) exclusive dealing.

84. Expensive, high-quality products that are purchased infrequently often reach consumers through a) selective distribution. b) highly-selective distribution. c) sole-source retailers. d) complex marketing channels. e) exclusive distribution.

e) exclusive distribution.

Expensive, high-quality products that are purchased infrequently often reach consumers through a) selective distribution. b) highly-selective distribution. c) sole-source retailers. d) complex marketing channels. e) exclusive distribution.

e) exclusive distribution.

74. The three major levels of intensity at which a company can choose to distribute its products are _____ distribution. a) narrow, medium, and wide b) cooperative, conflicting, and integrated c) intensive, extensive, and exclusive d) selective, cooperative, and conflicting e) exclusive, selective, and intensive

e) exclusive, selective, and intensive

The three major levels of intensity at which a company can choose to distribute its products are _____ distribution. a) narrow, medium, and wide b) cooperative, conflicting, and integrated c) intensive, extensive, and exclusive d) selective, cooperative, and conflicting e) exclusive, selective, and intensive

e) exclusive, selective, and intensive

55. Organizational buyers are especially partial to direct marketing channels when a) they buy cheap materials in large quantities. b) they try a new product for the first time. c) they are filling an order for a very important customer. d) a modified rebuy type of decision is involved. e) expensive and/or complex equipment is involved.

e) expensive and/or complex equipment is involved.

Organizational buyers are especially partial to direct marketing channels when a) they buy cheap materials in large quantities. b) they try a new product for the first time. c) they are filling an order for a very important customer. d) a modified rebuy type of decision is involved. e) expensive and/or complex equipment is involved.

e) expensive and/or complex equipment is involved.

97. Vertical channel integration a) results in two or more different management teams for each member of the channel. b) is made possible when a large corporation divests itself of smaller subsidiaries. c) is a shift back to the conventional channel of distribution. d) combines institutions at the same level of operation. e) is made possible by purchasing the operations of a link in the channel.

e) is made possible by purchasing the operations of a link in the channel.

Vertical channel integration a) results in two or more different management teams for each member of the channel. b) is made possible when a large corporation divests itself of smaller subsidiaries. c) is a shift back to the conventional channel of distribution. d) combines institutions at the same level of operation. e) is made possible by purchasing the operations of a link in the channel.

e) is made possible by purchasing the operations of a link in the channel.

Because of the significant investment many companies have in the products they sell to customers, they must develop and maintain adequate varieties of products to meet their customers' needs. This is called a) just-in-time. b) order processing. c) inventory management. d) merchandise stocking. e) logistical management.

e) logistical management.

73. When considering the best channel to use, all of the following are true with regard to larger firms except they a) can use an extensive product mix as a competitive tool. b) may be better able to negotiate better deals with vendors or other channel members. c) may have more distribution centers. d) may have the resources to develop their own sales force. e) may be better suited to serve customers in a particular region.

e) may be better suited to serve customers in a particular region.

When considering the best channel to use, all of the following are true with regard to larger firms except they a) can use an extensive product mix as a competitive tool. b) may be better able to negotiate better deals with vendors or other channel members. c) may have more distribution centers. d) may have the resources to develop their own sales force. e) may be better suited to serve customers in a particular region.

e) may be better suited to serve customers in a particular region.

130. Nathan is trying to decide which shipping company to use to transport his custom-made furniture to customers. Nathan is facing a decision about a) order entry. b) order completion. c) just-in-time. d) order handling. e) order delivery.

e) order delivery.

Nathan is trying to decide which shipping company to use to transport his custom-made furniture to customers. Nathan is facing a decision about a) order entry. b) order completion. c) just-in-time. d) order handling. e) order delivery.

e) order delivery.

116. As her Internet quilt and bed linen business grew, Danielle could no longer perform all the distribution activities, so she hired a shipper and an information technology firm to assist with these functions. Danielle is using _____ for physical distribution for her business. a) producers b) wholesalers c) retailers d) selling agents e) outsourcing

e) outsourcing

As her Internet quilt and bed linen business grew, Danielle could no longer perform all the distribution activities, so she hired a shipper and an information technology firm to assist with these functions. Danielle is using _____ for physical distribution for her business. a) producers b) wholesalers c) retailers d) selling agents e) outsourcing

e) outsourcing

113. The contracting of physical distribution tasks to third parties who do not have managerial authority within the marketing channel is known as a) illegal. b) logistics. c) warehousing. d) wholesaling. e) outsourcing.

e) outsourcing.

The contracting of physical distribution tasks to third parties who do not have managerial authority within the marketing channel is known as a) illegal. b) logistics. c) warehousing. d) wholesaling. e) outsourcing.

e) outsourcing.

Assuming everything else equal, the most cost-efficient way for British Petroleum to transport crude oil from remote oil drilling sites in central Alaska to shipping terminals on the Alaskan coast would be a) waterways. b) airways. c) tanker trucks. d) railroads. e) pipelines.

e) pipelines.

148. When companies operate their own facilities for storing and shipping products, these facilities are known as a) distribution centers. b) product storehouses. c) public warehouses. d) megawarehouses. e) private warehouses.

e) private warehouses.

When companies operate their own facilities for storing and shipping products, these facilities are known as a) distribution centers. b) product storehouses. c) public warehouses. d) megawarehouses. e) private warehouses.

e) private warehouses.

39. Consumers receive the benefits of place utility when a) they have to travel excessively to obtain products they want. b) retailers remain open 24 hours a day. c) they can stock up on products they need but not use them right away. d) they make purchases with credit and debit cards. e) products are available in locations where consumers want to buy them.

e) products are available in locations where consumers want to buy them.

Consumers receive the benefits of place utility when a) they have to travel excessively to obtain products they want. b) retailers remain open 24 hours a day. c) they can stock up on products they need but not use them right away. d) they make purchases with credit and debit cards. e) products are available in locations where consumers want to buy them.

e) products are available in locations where consumers want to buy them.

Using a freight forwarder usually a) is not economical for a small firm. b) increases transit time and shipping costs. c) increases transit time and sometimes lowers shipping costs. d) lowers shipping costs for a large firm. e) reduces transit time.

e) reduces transit time.

79. In marketing a new line of nursery furniture for infants and small children, the Fisher Company will most likely use ___________ distribution for the products. a) intensive b) exclusive c) horizontal d) priority e) selective

e) selective

In marketing a new line of nursery furniture for infants and small children, the Fisher Company will most likely use ___________ distribution for the products. a) intensive b) exclusive c) horizontal d) priority e) selective

e) selective

81. Durable goods such as television sets and DVD players generally reach their target markets through a) intensive distribution. b) channel cooperation. c) direct marketing. d) exclusive distribution. e) selective distribution.

e) selective distribution.

Durable goods such as television sets and DVD players generally reach their target markets through a) intensive distribution. b) channel cooperation. c) direct marketing. d) exclusive distribution. e) selective distribution.

e) selective distribution.

Channel decisions are important to marketers mostly because a) they are relatively flexible to change quickly. b) consumers value reasonable prices delivered through marketing channels. c) they dictate what promotional strategies companies should use. d) many businesses are marketing intermediaries. e) they involve long-term commitments and affect customer accessibility.

e) they involve long-term commitments and affect customer accessibility.

115. Physical distribution activities may be performed by a producer, wholesaler, retailer or a) not performed at all. b) the customer. c) investors in any of the previous three. d) solely through technology. e) they may be outsourced.

e) they may be outsourced.

Physical distribution activities may be performed by a producer, wholesaler, retailer or a) not performed at all. b) the customer. c) investors in any of the previous three. d) solely through technology. e) they may be outsourced.

e) they may be outsourced.

40. In a simple economy of five producers and five consumers, there would be ___________ transactions possible without an intermediary and _________ transactions possible with one intermediary. a) ten; twenty-five b) thirty; ten c) twenty-five; fifteen d) sixteen; eight e) twenty-five; ten

e) twenty-five; ten

119. According to the text, physical distribution cost tradeoffs enable firms to a) resolve pricing conflicts among channel partners. b) minimize risk during test marketing of new products. c) reduce costs of all distribution functions simultaneously. d) resolve pricing conflicts within industry sectors. e) utilize resources for greatest cost-effectiveness.

e) utilize resources for greatest cost-effectiveness.

99. Warner Bros. sells cookie jars, puzzles, photo albums, and other items featuring its popular cartoon characters directly through its own retail outlets. This is an example of a) channel leadership. b) channel cooperation. c) channel conflict. d) horizontal channel integration. e) vertical channel integration.

e) vertical channel integration.

Warner Bros. sells cookie jars, puzzles, photo albums, and other items featuring its popular cartoon characters directly through its own retail outlets. This is an example of a) channel leadership. b) channel cooperation. c) channel conflict. d) horizontal channel integration. e) vertical channel integration.

e) vertical channel integration.

Apple makes its computers available through its own stores, its website, and some major retailers. This is an example of Select one: a. vertical integration. b. horizontal integration. c. exclusive dealing. d. tying agreements. e. dual distribution.

e.

Manufacturers of convenience products such as chewing gum reach customers through thousands of retailers. What marketing channel are these manufacturers most likely to use? Select one: a. Producer, consumer b. Producer, wholesaler, agent, retailer, consumer c. Producer, retailer, consumer d. Retailer, consumer e. Producer wholesaler, retailer, consumer

e.

Piggyback, fishyback, and birdyback are terms usually associated with gaining efficiency in shipping through Select one: a. warehousing. b. packaging. c. lot sizes. d. transit time. e. containerization.

e.

When Zappos, Inc., an online marketer of shoes, accessories, and clothing, contracts out the physical distribution task to third parties such as UPS, this arrangement is called ___________. This is due to the fact that UPS does not have managerial authority within the Zappos marketing channel. Select one: a. warehousing. b. wholesaling. c. logistics. d. illegal. e. outsourcing.

e.

When three buyers purchase the products of three producers, nine transactions are required. If one intermediary serves both producers and buyers, the number of possible transactions is Select one: a. eighteen. b. twenty. c. five. d. fifteen. e. six.

e.

The most popular type of vertical marketing system is the _____________. a. corporate VMS b. administered VMS c. horizontal VMS d. vertical VMS e. contractual VMS

e. contractual VMS

Dell Computer and Harley Davidson are well known for their use of the JIT approach. To which of the following physical distribution activities does the JIT approach belong? a. transportation b. order processing c. warehousing d. materials handling e. inventory management

e. inventory management

Suppliers may use ________ to get rid of slow-moving inventory, while a franchiser may use them to maintain quality control and protect the franchiser's reputation a. refusal to deal b. exclusive dealing c. warehousing d. restricted sales territories e. tying agreements

e. tying agreements

The advantages of catalog retailing include ____________ and ____________ for customers because they do not have to visit a store.

efficiency; convenience

A commonly used computerized means of integrating order processing with production, inventory, accounting, and transportation is:

electronic data interchange

a commonly used computerized means of integrating order processing with production, inventory, accounting, and transportation is: -web-based inventory management -just-in-time -electronic data interchange -universal product codes -activity based management

electronic data interchange

honey farms is a maker of fine chocolates. the company's latest product, fudge-dipped strawberries, is the premier product in its fudge-dipped line. the product is very expensive and targeted to upscale consumers. which form of distribution would honey farms be likely to use for its new product: -intensive -selective -targeted -exclusive -premier

exclusive

Expensive, high-quality products that are purchased infrequently often reach consumers through:

exclusive distribution.

When a customer is considering the purchase of a product in a less-familiar product category, that individual is likely to rely more heavily on

external reference price

When a customer is considering the purchase of a product in a less-familiar product category, that individual is likely to rely more heavily on

external reference prices

Brand equity is a customer's favorable attitude toward a specific brand and, depending on strength, some likelihood of consistent purchase of the brand when needs for a product in this product category arise. a. True

false

Business customers prefer to deal with producers through intermediaries and thus eliminate much of the burden of sorting out details. a. True b. False

false

Factory outlet stores exist to compete with traditional retailers. a. True b. False

false

General-merchandise wholesalers are full-service merchant wholesalers who carry only a few product lines. a. True b. False

false

Industrial distributors and their agents are typical types of intermediaries found in channels for consumer products. a. True b. False

false

Labels should be a separate feature that can be detached from the package. ​ a. True b. False

false

Marketers frequently rely on personal selling to improve the effectiveness of other promotion mix elements, especially advertising and sales promotion. a. True b. False

false

The primary product offered by the department stores is convenience. a. True b. False

false

The receiver's response to the decoded message is called noise. a. True b. False

false

When a push policy is used in promoting a particular product, the producer promotes the product directly to consumers. a. True b. False

false

When encoding a message, the source should use signs or symbols that are new to the audience. a. True b. False

false

Luis operates a store called Fleet Feet Sports, which sells running apparel, shoes, and accessories. Luis enjoys operating Fleet Feet Sports, especially because he has ready access to management expertise and promotional support. He was able to open Fleet Feet Sports with limited capital. However, there are disadvantages Luis faces as well. For him, one of the biggest disadvantages is that he does not have complete control over his operation. Fleet Feet Sports is most likely a a. direct seller. b. wholesaler. c. producer. d. direct marketer. e. franchise.

franchise

Malcolm Lewis has come up with the idea of a system for picking up people's cars while they are at work, washing and waxing them, and returning them for a fee. Having been a big success in his home city, Malcolm plans to expand his operation into other cities. The service described here seems best suited to _____.

franchising

Logistics Plus is an organization that consolidates shipments from several industrial firms into more efficient lot sizes so that they can be more efficiently transported. Logistics Plus is an example of a(n) _____.

freight forwarder

Customers rely on _____ for product availability, breaking larger quantities into smaller ones, technical advice and service, financial assistance, and suitable assortments.

full-service wholesalers

The decision of Macy's to use even prices such as $60 for a Ralph Lauren Polo shirt is an application of odd-even pricing, and even prices are often used to

give a product an upscale or exclusive image

Pricing strategies and methods

help direct and structure the selection of a final price

A penetration pricing strategy is particularly appropriate when demand is

highly elastic

To expand the number of its retail outlets in the Washington, D.C. area, Whole Foods bought out a small chain of organic food stores in northern Virginia. This type of integration is called a. retail. b. vertical. c. upstream. d. forward. e. horizontal.

horizontal

to expand the number of its retail outlets in the washington dc area, whole foods bought out a small chain of organic food stores in northern virginia. this type of integration is called: -vertical -retail -horizontal -backward -forward

horizontal

Competitors' prices, along with the marketing variables they emphasize, are determining factors in

how important price will be to customers

Question Competitors' prices, along with the marketing variables they emphasize, are determining factors in Answer

how important price will be to customers.

After shopping in the same store for nearly two hours, Chelsea goes to Burger King for a Whopper, while Cynthia goes to the optical shop to see if her contacts are ready. They engage in these activities without leaving the store in which they have been shopping. They are most likely in a _____.

hypermarket

The result of an inventory stockout usually results in a(n)____.

increase in lost sales.

Wholesaler

individual or organization that facilitates and expedites wholesale transaction (sysco sells foods to schools, restaurants, hotels, hospital) -handles physical distribution of goods -furnishes channel information

Sales are most likely to have a direct relationship to product availability for products that use _____ distribution.

intensive

A price developed in the consumer's mind through experience with the product is called a(n)

internal reference price

For most products, a(n) ____ relationship exists between the price of a particular product and the quantity demanded.

inverse

A product is a price leader when

it is sold at less than cost in the hope that sales of other products will increase

A product is a price leader when

it is sold at less than cost in the hope that sales of other products will increase.

Which of the following is an advantage of franchising? a. Franchise agreement gives control to franchisor b. Franchise agreements are not uniform c. Limited capital is needed to start d. Franchisor dictates many aspects of business e. Franchisee pays for assistance from franchisor

limited capital is needed to start

Merchant wholesalers who provide very few marketing services and specialize in just a few functions are known as _____.

limited-service wholesalers

All of the following are examples of full-service wholesalers except:

mail-order wholesalers.

Immediately after the breakeven point, a firm starts to

make profits

An independent businessperson who is paid a commission to sell complementary products of different producers in an assigned territory without actually taking title of the merchandise is a(n):

manufacturers' agent

If Nabisco had established a pricing objective of selling one out of every three crackers consumed in the world, it would have established an objective based on

market share.

A cost-based pricing method commonly used in retail is called

markup pricing.

When considering the best channel to use, all of the following are true with regard to larger firms except they:

may be better suited to serve customers in a particular region.

Products such as light bulbs, canned soft drinks, and ice cream sandwiches are usually priced using

multiple-unit pricing

A limited-line wholesaler would be expected to carry a ____ product mix. a. wide and shallow b. narrow and shallow c. wide and deep d. wide e. narrow and deep

narrow and deep

Marketers at organizations engaged in nonprice competition

need competitive price information to make sure that their products are priced at approximately the same level as the prices of competing brands.

Marketers at organization engaged in nonprice competition

need competitive price information to make sure that their products are priced at approximately the same levels as the prices of competing brands

Maria recently put her house on the market at an asking price of $260,000. She realizes, however, that in order to sell the house, she may have to use

negotiated pricing

Maria recently put her house on the market at an asking price of $260,000. She realizes, however, that in order to sell the house, she may have to use

negotiated pricing.

Price skimming and penetration pricing are both strategies used for

new-product pricing.

Sellers that emphasize distinctive product features to encourage brand preferences among customers are practicing

non-price competition

Sellers that emphasize distinctive product features to encourage brand preferences among customers are practicing

nonprice competition

Lucas is planning a distribution strategy for his business. He is considering using direct marketing, direct selling, and automatic vending which are all examples of _____ retailing.

nonstore

Sophia is the new marketing manager for a new line of laptop, MP3, and smartphone accessories made by the company, Skull Candy. These accessories are trendy, colorful, and more expensive than others on the market. After considering the target market for the products, Sophia has decided not to use marketing-related activities conducted through telemarketing, and instead use vending, a type of _____.

nonstore retailing

Eliminating a wholesaler from a marketing channel will:

not eliminate the functions performed by that wholesaler.

Reference pricing is

o help price a firm's products. pricing a product at a moderate level and positioning it next to a more expensive model or brand.

Suppose that Rayban is considering a new line of sunglasses that would be sold in major department stores. The new line would be positioned as a more distinctive brand than the typical glasses sold through department stores, and would be priced higher than other brands in the store, but a lower price line than the current Rayban lines that are sold through more selective stores. In determining the price for this sunglass line, Rayban wants to gather information about all brands sold in department stores and about customers' perceptions of those brands. Reference: Ref 20-1 If Rayban selected the prices for its new sunglasses to be $60, $70, or $80, this would most likely be an example of using __________ pricing to enhance its distinctive positioning strategy.

odd-even

When Zappos Inc., an online marketer of shoes, accessories, and clothing, contracts out the physical distribution task to third parties such as UPS, this arrangement is known as _____. This is due to the fact that UPS does not have managerial authority within the Zappos marketing channel.

outsourcing

Marketers must take steps to make sure that the pricing objectives they set are consistent with the organization's ___________ objectives and ___________ objectives

overall; marketing

Marketers must take steps to make sure that the pricing objectives they set are consistent with the organization's ___ objectives and ___ objectives

overall; marketing

The management at Allied Electronics is having difficulty in raising the introductory price on system components to cover the increased costs of producing the sensing devices for home security systems. Apparently, Allied used a(n) ___________ strategy in pricing these components.

penetration

The management at Allied Electronics is having difficulty in raising the introductory price on system components to cover the increased costs of producing the sensing devices for home security systems. Apparently, Allied using a(n) ___ strategy in pricing these components

penetration

All of the following are pricing strategies used by companies establishing prices of multiple products within a product line except: -premium pricing -price lining -captive pricing -bait pricing -penetration pricing

penetration pricing

Which of the following would be used in setting the price of a new product if considerable competition is expected?

penetration pricing.

A problem associated with ___ is that consumers can predict when prices will be lowered and delay purchases until that time

periodic discounting

A problem associated with _____ is that consumers can predict when prices will be lowered and delay purchases until that time.

periodic discounting

The "White Sale" that many department stores have every year a few weeks after Christmas is an example of

periodic discounting

The "White Sale" that many department stores have every year a few weeks after Christmas is an example of

periodic discounting

Assuming everything else equal, the most cost-efficient way for British Petroleum to transport crude oil from remote oil drilling sites in central Alaska to shipping terminals on the Alaskan coast would be:

pipelines.

A product that has more features than those of its competition, or that is perceived to be higher quality, warrants using which type of pricing strategy?

premium pricing

If REVO sets the price for its sunglasses at $240, it is using psychological pricing to convey

prestige

If REVO sets the price for its sunglasses at $240, it is using psychological pricing to convey

prestige.

Most flexible variable in the marketing mix

price

When marketers emphasize price as an issue and match or beat the prices of other companies, they are using

price competition

A measure of sensitivity of demand in relation to changes in price is

price elasticity of demand

To attract customers into a store, Safeway advertises its milk at less than cost, hoping that customers will purchase other groceries as well. This pricing strategy is called

price leader pricing.

When an organization sets a number of prices for selected groups of merchandise, this is commonly referred to as

price limiting

All of the following are psychological techniques except -customary pricing -prestige pricing -reference pricing -odd-even pricing -price skimming

price skimming

If Norelco introduced a new electric razor that sonically removes hair and priced is first at $175 and then at $150 before reducing the price to $100, the firm's initial pricing strategy is known as

price skimming

When business charge the highest possible price that customers who really want the new product will pay, they are using

price skimming

If Norelco introduced a new electric razor that sonically removes hair and priced it first at $175 and then at $150 before reducing the price to $100, the firm's initial pricing strategy is known as

price skimming.

Question When Sharp first introduced its line of graphing calculators, it set the price quite high; it has lowered the price as competitors have entered the market. The pricing strategy initially used by Sharp is called

price skimming.

When businesses charge the highest possible price that customers who really want the new product will pay, they are using

price skimming.

A danger associated with engaging in price competition is that competitors can also change prices quickly and aggressively, which can result in a _______ that will be harmful to both companies

price war

Isabella is a product manager for The Container Store, a retailer of plastic bins and other storage containers. Sales have been declining in the past nine months and her management team is pressuring her to compete based on price discounts. However, Isabella is aware of the dangers associated with engaging in price competition. She knows that competitors can also change prices quickly and aggressively, which can result in a(n) ____ that will be harmful to both companies.

price war

Reference pricing is

pricing a product at a moderate level and positioning it next to a more expensive model or brand

If an organization sets prices to recover research and development expenses and establish a premium quality image for its product, it would be using a ___ pricing objective

product quality

Question If an organization sets prices to recover research and development expenses and establish a premium quality image for its product, it would be using a _________ pricing objective.

product quality

The pricing of Clinique make up considerably higher than brands such as Cover Girl, Revlon, and Maybelline is used to communicate ___, which is the company's primary pricing objective

product quality

The pricing of Clinique makeup considerably higher than brands such as Cover Girl, Revlon, and Maybelline is used to communicate ______, which is the company's primary pricing objective.

product quality

Consumers receive the benefits of place utility when:

products are available in locations where consumers want to buy them.

What type of pricing strategy is used in situation where demand for a product is price inelastic and the seller has an ethical responsibility not to overcharge the client

professional pricing

Westin Inc. has an objective of achieving a 25 percent return from its overall sales. This is an example of a ______ pricing objective.

profit

Westin Inc. has an objective of achieving a 25 percent return from its overall sales. This is an example of a ___ pricing objective

profit

When a company attempts to influence a consumer's perception of price to make a product's price more attractive and reduce "sticker shock," it is using a ______ pricing strategy

psychological

When a company attempts to influence a consumer's perception of price to make a product's price more attractive and reduce "sticker shock," it is using a ___ pricing strategy.

psychological

For customers, value is function of the product's

quality attributes

If Wilson Sporting Goods faces a standard demand curve that exists for most products, as it raises the price of its tennis rackets, the

quantity demanded goes down

A marketer is most likely to set prices according to a cash-flow objective when a

quick return on investment is desired

A marketer is most likely to set prices according to a cash-flow objective when a

quick return on investment is desired.

Fiona is a wholesaler who buys greeting cards from Hallmark Cards Inc. and resells them by maintaining displays of greeting cards in drugstores and discount stores. Her daily activities include straightening the cards, pulling outdated or slow sellers, and installing new cards. Fiona is called a(n) _____.

rack jobber

If a business decides to reduce its prices once in a while on unsystematic basis, it is using

random discounting

when sophie buys organic produce for her household using a channel with only one intermediary, that intermediary is classified as a: -retailer -wholesaler -broker -functional middleman -producer

retailer

Most pricing objectives based on ___ are achieved by trial and error because not all cost and revenue data are available when prices are set

return on investment

Most pricing objectives based on ___________ are achieved by trial and error because not all cost and revenue data are available when prices are set.

return on investment

The management at Kohler Inc., a manufacturer of sinks, tubs, and other plumbing products, thinks that there will soon be a major growth in home remodeling due to rising interest rates. The management decides to use _____ in several key cities to provide support services for its sales force in those areas, carry inventory, and offer credit and other services to its retail plumbing customers.

sales branches

Abbot Laboratories offers ceramic coffee mugs to its physician customers when it promotes a new drug. This example illustrates Abbot's use of which one of the following elements of the promotion mix? a. Personal selling b. Packaging c. Public relations d. Social marketing e. Sales promotion

sales promotion

When McDonald's uses the Monopoly game in which customers receive game pieces with each visit and try to assemble a set of properties to win prizes, McDonald's is utilizing a. personal selling. b. advertising. c. sales promotion. d. guerrilla marketing. e. public relations.

sales promotion

Executives in Japan decided to price Lexus luxury cars in the United States at $55,000 while pricing them at $66,000 in their own country. This is an example of

secondary-market pricing.

When Merrell introduced its Chameleon hiking shoes, they were available through REI, Dick's, Bass Pro Shops, and a few other retailers. The Merrell Chameleon hiking shoe was most likely distributed through the _____ of distribution. a. intensive level b. agent form c. extensive level d. selective level e. exclusive level

selective level

A channel member that markets all of a manufacturer's goods, has authority over price, promotion, and distribution, but does not take title to the product is a a. manufacturers' agent. b. broker. c. selling agent. d. general-line wholesaler. e. commission merchant.

selling agent

Markup is measured either as a percentage of ___ or a percentage of ___

selling price; cost

The ___________ prohibits price fixing among firms in an industry.

sherman antitrust act

fragile products that require special handling are more likely to be distributed through: -longer channels -shorter channels -direct channels -strategic alliances -exclusive outlets

shorter channels

In contrast to business purchases, consumer purchases are likely to be influenced by _____.

social influences

A sale at The Bon Marche the day after Thanksgiving to kick off the Christmas season would be considered

special-event pricing

Question A sale at The Bon Marche the day after Thanksgiving to kick off the Christmas season would be considered

special-event pricing.

Maintaining a certain market share, meeting competitors' prices, maintaining a favorable image, and achieving price stability are all associated with a ___ pricing objective

status quo

Question Maintaining a certain market share, meeting competitors' prices, maintaining a favorable image, and achieving price stability are all associated with a _____ pricing objective.

status quo

What type of pricing objective would an organization use if it were in a favorable position and desired nothing more?

status quo

Which pricing objective de-emphasizes price and can lead to a climate of nonprice competition in an industry?

status quo

Which type of pricing objective can reduce a firm's risk by helping to stabilize demand for its products?

status quo

walmart is working with its suppliers, using tools such as electronic billing, purchase order verification, and bar code technology, to integrate data used to improve overall performance. this is an example of: -supply chain management -a vertical marketing system -a horizontal marketing system -channel conflict -dual distribution

supply chain management

Greenmart Corp. is working with its suppliers, using tools such as electronic billing, purchase order verification, and bar code technology, to integrate data used to improve overall performance. This is an example of:

supply-chain management.

Running a bike sale in order to generate enough cash flow to pay creditors is typical in a situation in which a firm's primary pricing objective is

survival

When a company adjusts price levels so that it can increase sales volume to levels that match the organization's expenses, it is said to employ a _________ objective.

survival

When a company adjusts price levels so that it can increase sales volume to levels that match the organization's expenses, it is said to employ a ___ objective

survival

false

t or f: It is usually easy to obtain an accurate price list for a competitor's products.

False

t or f: Setting prices for business customers is very similar to setting prices for consumers.

true

t or f: ​Noncumulative discounts are one-time reductions in prices based on the number of units purchased, the dollar value of the order, or the product mix purchased.

Selling agents perform every wholesaling activity except _____.

taking title to products

When products are presented to television viewers, who can purchase them by calling a toll-free number and paying with a credit card, _____ is being used.

television home shopping

A sale that advertised prices "up to 65% off" the original prices uses

tensile pricing

The point at which the costs of producing a product equal the revenue earned from selling the product is

the breakeven point

Price is a key element in the marketing mix because it relates directly to

the generation of total revenue.

A price-skimming strategy assumes that

the initial demand is highly inelastic

At the breakeven point,

the money a company brings in from selling products equals the amount spent producing the products

Which of the following is a requirement for setting pricing objectives?

the objectives should be explicitly stated

Order processing is defined as:

the receipt and transmission of sales order information.

Price is

the value that is exchanged for products in a marketing transaction

Price is

the value that is exchanged for products in a marketing transaction.

Channel decisions are important to marketers mostly because:

they involve long-term commitments and affect customer accessibility.

Marketers improve their ability to establish prices appropriately when

they know prices charged for competing brands

Marketers improve their ability to establish prices appropriately when

they know prices charged for competing brands.

Bundle pricing may be perceived to be of value by customers because

they prefer buying a combination of bundled products in a single transaction, which saves time, effort, and perhaps money.

Bundle pricing may be perceived to be of value by customers because

they prefer buying a combination of bundled products in a single transaction, which saves time, effort, and perhaps money

The main reason a manufacturer will prohibit intermediaries from selling its products outside designated sales territories is to:

tighten its control over distribution of its products.

If a product has an inelastic demand and the manufacturer raises its price,

total revenue will increase

A store that offers a wide variety of shoes for men, women, and children would most likely be considered a(n) _____.

traditional specialty retailer

Wholesaling

transactions in which products are bought for resale, for making other products, or for general business operations.

A communications channel is the vehicle used to transmit the coded message from the source to the receiver. a. True b. False

true

Advertising is a paid form of nonpersonal communication. a. True b. False

true

Appliances are an example of a major product line commonly carried by discount stores. a. True b. False

true

Direct marketing is the selling of products outside the confines of a retail facility. a. True b. False

true

Marketing intermediaries bridge the gap between suppliers (or producers) and buyers (or consumers). a. True b. False

true

Personal selling tends to cost more than advertising. a. True b. False

true

Public relations is a broad set of communication efforts used to create and maintain favorable relationships between the organizations and its stakeholders. a. True b. False

true

By buying in large quantities and delivering to customers in smaller lots, a wholesaler may perform all of the following physical distribution activities except:

unit pricing.

Spam

unsolicited email

Competition-based pricing is

used when costs and revenues are secondary to competitors' prices

When consumers are making do with less expensive products and shopping more selectively, manufacturers and retailers must focus on the ___ of their products

value

Disney sells apparel, cookie jars, puzzles, photo albums, and other items featuring its popular characters directly through its own retail outlets. This is an example of a. channel cooperation. b. vertical channel integration. c. horizontal channel integration. d. channel leadership. e. channel conflict.

vertical channel integration

benetton, a leading manufacturer of knitwear in italy, expanded its operations to include retail outlets in the united states and south america. this type of channel is called: -horizontal integration -vertical integration -multilevel integration -retail integration -merchandising integration

vertical integration

Old Spice created a series of online videos on YouTube through its "The Man Your Man Could Smell Like" campaign in which the main character responded to viewer questions. These entertaining videos were shared by thousands of people, spreading the message created by Old Spice. This is an example of a. advertising. b. sales promotion. c. viral marketing. d. public relations e. personal selling.

viral marketing

Retail facilities located in big, low-cost buildings with large on-premise inventories and minimal services are called _____.

warehouse showrooms

Marginal analysis involves examining

what happens to a firm's costs and revenues when production is changed by one unit.

Which of the following are personal informal exchanges of communication that customers share with one another about products, brands, and companies? a. Word-of-mouth communication b. Guerrilla communication c. Viral marketing d. Online chat room communication e. Buzz marketing

word- of- mouth communication

66. What is a primary difference between an industrial distributor and a manufacturers' agent? a) A manufacturers' agent does not acquire title nor usually take possession of the products whereas an industrial distributor does. b) A manufacturers' agent is employed by the manufacturers while an industrial distributor is independent. c) An industrial distributor is employed by the manufacturers while a manufacturers' agent is independent. d) A manufacturers' agent rarely adds any value to the marketing channel while an industrial distributor reduces costs significantly. e) An industrial distributor does not form relationships with customers for repeat business whereas a key asset of a manufacturers' agent is his knowledge of his customers.

a) A manufacturers' agent does not acquire title nor usually take possession of the products whereas an industrial distributor does.

Which of the following is a large-scale, members-only, retailer that combines cash-and-carry wholesaling with discount retailing? a. Warehouse showroom b. Warehouse club c. Catalog showroom d. Category killer e. Hypermarket

b

Manufacturers of convenience products such as chewing gum reach customers through thousands of retailers. What marketing channel are these manufacturers most likely to use? a) Producer, consumer b) Producer, wholesaler, retailer, consumer c) Producer, wholesaler, agent, retailer, consumer d) Producer, retailer, consumer e) Retailer, consumer

b) Producer, wholesaler, retailer, consumer

The Diamond Salt Company needs to transport 20 tons of salt from Baton Rouge, Louisiana, to Cincinnati. Which of the following methods would be the cheapest for transporting this cargo? a) Pipelines b) Waterways c) Trucks d) Airways e) Railroads

b) Waterways

At the annual managers' planning conference, Jackie Conrad asks the other managers to consider her proposal to increase the quantity of inventory to a five-week supply in order to increase the percentage of completely filled customer orders from 85 percent to 90 percent. She states that she is willing to trade off the ___________ incurred for the positive effect of better customer service. a) lower response time b) higher warehousing costs c) higher transportation costs d) higher order processing costs e) higher performance levels

b) higher warehousing costs

126. By receiving orders online instead of through a paper-based ordering system, Barnes and Nobles has been able to save time and money on a) electronic data interchange. b) order entry. c) inventory management. d) order delivery. e) order handling.

b) order entry.

By receiving orders online instead of through a paper-based ordering system, Barnes and Nobles has been able to save time and money on a) electronic data interchange. b) order entry. c) inventory management. d) order delivery. e) order handling.

b) order entry.

Possession utility is best described as a) products being available in places where the customers wish to purchase them. b) the customer having access to the product to use now or store and use later. c) having a company's products available when a customer needs them. d) being able to legally own a product despite restrictions on trade. e) getting the products to the consumers in as short of time as possible for ownership.

b) the customer having access to the product to use now or store and use later.

When Mia and Shane are planning their honeymoon, their travel agent tells them that if they buy a special package, their trip to Paris will include meals, tickets to the theater, and a rental car in addition to airfare and a hotel. This is an example of the use of

bundle pricing.

Large, self-service stores that carry a complete line of food products and some nonfood products are called a. discount stores. b. superstores. c. supermarkets. d. department stores. e. warehouse clubs.

c

135. What is the reorder point if the usage rate is 10 units per day, the order lead time is five days, and the safety stock is 30? a) 50 b) 150 c) 80 d) 200 e) 400

c) 80

What is the reorder point if the usage rate is 10 units per day, the order lead time is five days, and the safety stock is 30? a) 50 b) 150 c) 80 d) 200 e) 400

c) 80

60. Which of the following describes a disadvantage of using industrial distributors? a) Industrial distributors possess considerable market information. b) Their marketing exchange relationships are very focused. c) They are unlikely to handle bulky items or items that are slow sellers. d) Industrial distributors sell specific brands aggressively. e) Industrial distributors acquire title to the products and take possession.

c) They are unlikely to handle bulky items or items that are slow sellers.

Which of the following describes a disadvantage of using industrial distributors? a) Industrial distributors possess considerable market information. b) Their marketing exchange relationships are very focused. c) They are unlikely to handle bulky items or items that are slow sellers. d) Industrial distributors sell specific brands aggressively. e) Industrial distributors acquire title to the products and take possession.

c) They are unlikely to handle bulky items or items that are slow sellers.

Customers who purchase computer laptops from manufacturer websites are acquiring products through a) the most efficient channel of distribution. b) the most common type of marketing channel. c) a direct-marketing channel. d) a business-to-business channel of distribution. e) an indirect-marketing channel.

c) a direct-marketing channel.

96. If a wholesaler continually emphasizes and promotes one company's products over a competing company's products, _____ is likely to result. a) channel cooperation b) vertical channel integration c) channel conflict d) horizontal channel integration e) channel leadership.

c) channel conflict

117. The main objective of physical distribution should be to a) decrease costs while increasing market coverage. b) increase service and market coverage. c) decrease costs while increasing service. d) increase market coverage and channel power. e) balance costs and market coverage.

c) decrease costs while increasing service.

Jeff Wood's company buys machine tools from large producers and sells them to several Midwestern manufacturing companies. The company Jeff works for carries inventories of the tools, which reduces capital requirements for the producers. Jeff's company is an example of a(n) ___________ in a distribution channel. a) direct distributor b) manufacturers' agent c) industrial distributor d) producers' agent e) wholesalers' agent

c) industrial distributor

43. Without wholesalers and other intermediaries, a) most products would be much less expensive because fewer companies would be handling the product. b) products would be cheaper because the functions of intermediaries would be eliminated. c) products would likely be more expensive due to the use of less efficient channel members. d) products would never be able to make it to the ultimate consumer at any price without passing through intermediaries. e) many products would be more expensive because retailers would expect more profit.

c) products would likely be more expensive due to the use of less efficient channel members.

A single leader who controls and organizes a marketing channel is known as a:

channel captain.

Calhoun Produce, a _____, takes possession of truckloads of tomatoes, arranges for storage, and transports them to auctions to be sold.

commission merchant

When determining markup as a percentage of cost, divide the markup amount by

cost

What are the three primary tasks of order processing? a) Order receipt, order delivery, order follow-up b) Order handling, inventory management, order delivery c) Materials handling, warehousing, order delivery d) Order handling, order entry, order delivery e) Order receipt, order checking, order delivery

d) Order handling, order entry, order delivery

Nationally distributed consumer convenience products are most likely distributed through which of the following channels? a) Producer, consumers b) Producer, agents, wholesalers, retailers, consumers c) Producer, wholesalers, consumers d) Producer, wholesalers, retailers, consumers e) Producer, industrial distributor, wholesalers, retailers, consumers

d) Producer, wholesalers, retailers, consumers

Which of the following physical distribution functions involves design and operation of facilities for storing goods? a) Order processing b) Materials handling c) Transportation d) Warehousing e) Inventory management

d) Warehousing

147. Morgan Steel Company leases a warehouse in Jacksonville, Alabama to serve Deep South markets that were large enough and stable enough to make a long-term commitment to fixed facilities. This is a a) flexible warehouse. b) dispatching center. c) distribution center. d) private warehouse. e) public warehouse.

d) private warehouse.

Morgan Steel Company leases a warehouse in Jacksonville, Alabama to serve Deep South markets that were large enough and stable enough to make a long-term commitment to fixed facilities. This is a a) flexible warehouse. b) dispatching center. c) distribution center. d) private warehouse. e) public warehouse.

d) private warehouse.

149. Air-Tech management learns that last month's production run of heaters at its Newark plant is defective and announces a recall of the specific models produced. To handle the large number of products it expects to receive for replacement of the defective thermostats, Air-Tech will probably need to add a ___________ to its distribution network. a) distribution center b) private warehouse c) dispatching shipment system d) public warehouse e) materials handling system

d) public warehouse

Air-Tech management learns that last month's production run of heaters at its Newark plant is defective and announces a recall of the specific models produced. To handle the large number of products it expects to receive for replacement of the defective thermostats, Air-Tech will probably need to add a ___________ to its distribution network. a) distribution center b) private warehouse c) dispatching shipment system d) public warehouse e) materials handling system

d) public warehouse

156. Magnetic Springs wants to ship its bottled water with the most flexible schedules and routes of all transportation modes. Magnetic Springs should use ___________ as its transportation mode. a) railroads b) waterways c) pipelines d) trucks e) airways

d) trucks

Magnetic Springs wants to ship its bottled water with the most flexible schedules and routes of all transportation modes. Magnetic Springs should use ___________ as its transportation mode. a) railroads b) waterways c) pipelines d) trucks e) airways

d) trucks

The supply chain includes Select one: a. suppliers, producers, intermediaries, and customers. b. suppliers and suppliers' suppliers. c. buyers, seller, marketing intermediaries, and agents. d. all entities that facilitate product distribution. e. producers, wholesalers, and retailers.

d.

The three primary bases for developing prices are

demand, competition, and cost

The three primary bases for developing prices are

demand, competition, and cost.

If General Motors determines that it wants to sell 200,000 Chevrolet Acadias and sets the price at $29,500 because it knows that at that price it will reach that goal, the firm would be using a ___________ pricing method.

demand-based

If General Motors determines that it wants to sell 200,000 Chevrolet Acadias and sets the price at $29,500 because it knows that at that price it will reach that goal, the firm would be using a ____ pricing method.

demand-based

During July and August, Lakewood Links Golf Course, located in S.C., offers weekday rates of $13 for a round of golf with a cart. During the rest of the year, the weekday rates are between $25 and $35. This is an example of the use of

demand-based pricing

The fact that senior citizens are charged a lower price at movie theatres than younger adults is an example of ___ pricing

differential

Pricing the basic product in a product line low while pricing related items at a higher level is called Answers: a. premium pricing. b. price lining. c. price skimming. d. bait pricing. e. captive pricing.

e

Retailers that sell only to members and feature discount retailing combined with cash-and carry wholesaling are called a. hypermarkets. b. retail groups. c. warehouse showrooms. d. discount stores. e. warehouse clubs.

e

163. What two modes of transportation are used when containers are shipped by piggyback? a) Railroads and airways b) Trucks and airways c) Pipelines and trucks d) Waterways and railroads e) Railroads and trucks

e) Railroads and trucks

What two modes of transportation are used when containers are shipped by piggyback? a) Railroads and airways b) Trucks and airways c) Pipelines and trucks d) Waterways and railroads e) Railroads and trucks

e) Railroads and trucks

Dell Computers is a nationally recognized manufacturer of computers for the small business and home markets. If Dell were to order one of its wholesalers not to carry any computer products other than those made by Dell, this arrangement would be called a) a tying contract. b) refusal to deal. c) a restricted sales territory. d) a restricted channel. e) exclusive dealing.

e) exclusive dealing.

160. Assuming everything else equal, the most cost-efficient way for British Petroleum to transport crude oil from remote oil drilling sites in central Alaska to shipping terminals on the Alaskan coast would be a) waterways. b) airways. c) tanker trucks. d) railroads. e) pipelines.

e) pipelines.

166. Using a freight forwarder usually a) is not economical for a small firm. b) increases transit time and shipping costs. c) increases transit time and sometimes lowers shipping costs. d) lowers shipping costs for a large firm. e) reduces transit time.

e) reduces transit time.

34. Channel decisions are important to marketers mostly because a) they are relatively flexible to change quickly. b) consumers value reasonable prices delivered through marketing channels. c) they dictate what promotional strategies companies should use. d) many businesses are marketing intermediaries. e) they involve long-term commitments and affect customer accessibility.

e) they involve long-term commitments and affect customer accessibility.

In a simple economy of five producers and five consumers, there would be ___________ transactions possible without an intermediary and _________ transactions possible with one intermediary. a) ten; twenty-five b) thirty; ten c) twenty-five; fifteen d) sixteen; eight e) twenty-five; ten

e) twenty-five; ten

According to the text, physical distribution cost tradeoffs enable firms to a) resolve pricing conflicts among channel partners. b) minimize risk during test marketing of new products. c) reduce costs of all distribution functions simultaneously. d) resolve pricing conflicts within industry sectors. e) utilize resources for greatest cost-effectiveness.

e) utilize resources for greatest cost-effectiveness.

When a producer, wholesaler, or retailer has the ability to influence another channel member's goal achievement, it ____________. a. is vertically integrated b. is multichannel distribution c. is horizontally integrated d. is a channel captain e. has channel power

e. has channel power

Some companies attempt to reduce or eliminate the use of frequent short-term price reductions by using

everyday low prices.

The major levels of intensity at which a company can choose to distribute its products are _____distribution.

exclusive, selective, and intensive

Advertising makes possible immediate feedback from consumers. a. True b. False

false

Retailers form an important link in the marketing channel because they are both producers and customers. a. True b. False

false

Wholesaling is the final part of the marketing process in which exchanges are entered into for personal, family, or household purposes. a. True b. False

false

Ford sells the Ford Mustang, Ford F-150, and the Ford Explorer. This is an example of a. individual branding. b. shared naming. c. family branding. d. single branding. e. brand suspension.

family branding

Your friend Elias comes to you for advice because of your amazing business prowess. He would like to start his own business, but he is concerned about the high failure rate. If possible, he would like to mitigate his risk somewhat. He also has limited capital and not much business experience. You recommend that Elias consider opening a ____, but you warn him that one disadvantage to doing so is that he would have ____. a. franchise; less control b. wholesaler; more debt c. retailer; less control d. direct sales organization; limited promotional opportunities e. wholesaler; less time

franchise; less control

A penetration pricing strategy is particularly appropriate when demand is

highly elastic.

A cost-based pricing method commonly used in retail is called

mark-up prices

If Nabisco had established a pricing objective of selling one out of every three crackers consumed in the world, it would have established an objective based on

market share

A significant factor in the rise of multichannel retailing is __________ retailing.

online

If Nabisco wants to quickly gain a large market share with its new line of reduced-fat snack crackers, it should use

penetration pricing

All of the following are pricing strategies used by companies establishing prices of multiple products within a product line excep

penetration pricing.

Glenwood Pet Hospital is considering implementing a new pricing strategy for its veterinarian services. After reviewing the previous three years' revenue, Glenwood finds that most of its customers bring their pets in for the required annual vaccinations and then only if the animal is ill. Glenwood's objective is to generate more income per customer on an annual basis. The hospital has previously priced its services by charging a flat fee for the office visit, a fee for each vaccine, and a fee for each type of examination beyond the basic office visit. Most customers pay the flat office fee and a fee for a rabies vaccine. Glenwood is now considering a new plan where the pet owner would pay one fee that would cover an office visit, the required rabies vaccine, and additional vaccines that prevent heartworm, kennel-cough, and fleas. Glenwood hopes to encourage the pet owners to view their pet's health as part of a prevention program, rather than a one-time annual visit. Reference: Ref 20-2 Glenwood's closest competitor, The Hearthstone Pet Hospital, currently charges $60 for each basic office visit. If Glenwood were to price its basic office visit at $45, it would most likely be employing which of the following?

penetration pricing.

Glenwood Pet Hospital is considering implementing a new pricing strategy for its veterinarian services. After reviewing the previous three years' revenue, Glenwood finds that most of its customers bring their pets in for the required annual vaccinations and then only if the animal is ill. Glenwood's objective is to generate more income per customer on an annual basis. The hospital has previously priced its services by charging a flat fee for the office visit, a fee for each vaccine, and a fee for each type of examination beyond the basic office visit. Most customers pay the flat office fee and a fee for a rabies vaccine. Glenwood is now considering a new plan where the pet owner would pay one fee that would cover an office visit, the required rabies vaccine, and additional vaccines that prevent heartworm, kennel-cough, and fleas. Glenwood hopes to encourage the pet owners to view their pet's health as part of a prevention program, rather than a one-time annual visit. Reference: Ref 20-2 Glenwood has decided that it is going to offer a special package offer if the prevention plan is purchased within the first 30 days of each year's time for vaccinations. This type of pricing strategy would be an example of

periodic discounting.

Which of the following pricing strategies often results in a retailer losing money on the product?

price leader

Which of the following pricing strategies often results in a retailer losing money on the product?

price leader

To attract customers into a store, Safeway advertises its milk at less than cost, hoping that customers will purchase other groceries as well. This pricing strategy is called

price leader pricing

When Sharp first introduced its line of graphing calculators, it set the price quite high; it has lowered the price as competitors have entered the market. The pricing strategy initially used by Sharp is called

price skimming

Question Suppose that Rayban is considering a new line of sunglasses that would be sold in major department stores. The new line would be positioned as a more distinctive brand than the typical glasses sold through department stores, and would be priced higher than other brands in the store, but a lower price line than the current Rayban lines that are sold through more selective stores. In determining the price for this sunglass line, Rayban wants to gather information about all brands sold in department stores and about customers' perceptions of those brands. Reference: Ref 20-1 Given Rayban's plan for positioning the new sunglass line, they should use a __________ strategy when introducing their new product.

price-skimming

Research indicates that both market share and ___ are good indicators of profitability

product quality

Research indicates that both market share and ___________ are good indicators of profitability.

product quality

consumers receive the benefits of place utility when: -they have to travel excessively to obtain products they want -retailers remain open 24 hours a day -they can stock up on products they need but not use them right away -they make purchases with credit and debit cards -products are available in locations where consumers want to buy them

products are available in locations where consumers want to buy them

Price leaders, comparison discounting, and special-event pricing are applications of

promotional pricing

Price leaders, comparison discounting, and special-event pricing are applications of

promotional pricing.

Dawn is concerned that there are not enough buyers in Centerville to support her Quilts Unlimited store. Her business adviser suggests that she should not limit her promotion to the Centerville population but also consider out shoppers. This approach is most likely to work if her store is located in a:

regional shopping center

Webrooming is ______________________.

reseaching products online and then head to the nearest store to make the puchase

Eric is concerned about not having enough air conditioning units in inventory during June, July, and August so he keeps extra inventory to guard against stockouts during this critical period. Eric is keeping:

safety stock

Matt's girlfriend tells him she wants a cashmere sweater for Christmas. Matt decides to go to a store that provides the best possible selection of sweaters. His best choice would be to shop at a _____ store.

traditional specialty

Sometimes a marketing channel is quite simple and short, perhaps running directly from a producer to the final user or consumer. a. True b. False

true

The cost of a business product may affect the distribution channel selected for it. a. True b. False

true


Kaugnay na mga set ng pag-aaral

End of Chapter Quiz: Florida Statutes, Rules and Regulations Pertinent to Life Insurance

View Set

Chapter 2- Signs, Signals and Roadway Markings

View Set

Top 100 referenced books in NAQT question sets

View Set

2. Legal concepts of the Insurance contract

View Set

Chapter 17 Program Design, Ch 16: Exercise technique for alternative modes and nontraditional implement training, Ch.15 quiz, Ch.14 quiz

View Set

Ch 50: Nursing Care of a Family when a Child has a Vision or Hearing Disorder

View Set

Social Studies- American History 1127 Self Test Study

View Set